diktat teori bilangan minggu3-13

86
BAB II KETERBAGIAN 2.1 Pendahuluan Pada pertemuan minggu ke-3, dan 4 ini dibahas konsep keterbagian, algo- ritma pembagian dan bilangan prima pada bilangan bulat. Relasi keterbagian pada himpunan semua bilangan bulat memunculkan banyak sifat menarik. Dari relasi ini dapat didefinisikan pengertian hasil bagi dan sisa pembagian, sehingga dapat membangkitkan operasi pembagian bilangan bulat dan konsep modulo. Dengan mempelajari bab ini, diharapkan: 1. Mahasiswa bisa memahami pengertian keterbagian. 2. Mahasiswa bisa mengidentifikasi bilangan prima 3. Mahasiswa bisa menjelaskan pengertian algoritma pembagian 4. Mahasiswa bisa menerapkan sifat-sifat keterbagian dan algoritma pemba- gian pada masalah bilangan bulat 2.2 Keterbagian Sejak di sekolah dasar telah dikenal beberapa operasi pada bilangan bulat, diantaranya penjumlahan(+), pengurangan(), perkalian(× atau ·) dan pembagian(: atau /). Untuk sebarang dua bilangan bulat berlaku jumlah, selisih dan hasil kalinya masing-masing merupakan bilangan bulat, tetapi pembagian bilangan yang satu dengan yang lain belum tentu merupakan bilangan bulat. Definisi 2.2.1. Diberikan bilangan bulat m dan n (n ̸=0). Bilangan m dikatakan habis dibagi oleh n atau n membagi m, ditulis n|m, jika terdapat bilangan bulat k dengan sifat m = kn. Jika m habis dibagi oleh n, maka m disebut kelipatan dari n dan n disebut pembagi atau faktor dari m. Jika m tidak habis dibagi oleh n, dituliskan n ̸ | m. Karena 0 = 0.n, diperoleh bahwa n|0 untuk setiap bilangan bulat n. Sebaliknya, 0 ̸ | m untuk 11

Upload: abinailah

Post on 12-Dec-2015

190 views

Category:

Documents


4 download

DESCRIPTION

bilangan

TRANSCRIPT

BAB II

KETERBAGIAN

2.1 Pendahuluan

Pada pertemuan minggu ke-3, dan 4 ini dibahas konsep keterbagian, algo-

ritma pembagian dan bilangan prima pada bilangan bulat. Relasi keterbagian

pada himpunan semua bilangan bulat memunculkan banyak sifat menarik. Dari

relasi ini dapat didefinisikan pengertian hasil bagi dan sisa pembagian, sehingga

dapat membangkitkan operasi pembagian bilangan bulat dan konsep modulo.

Dengan mempelajari bab ini, diharapkan:

1. Mahasiswa bisa memahami pengertian keterbagian.

2. Mahasiswa bisa mengidentifikasi bilangan prima

3. Mahasiswa bisa menjelaskan pengertian algoritma pembagian

4. Mahasiswa bisa menerapkan sifat-sifat keterbagian dan algoritma pemba-

gian pada masalah bilangan bulat

2.2 Keterbagian

Sejak di sekolah dasar telah dikenal beberapa operasi pada bilangan bulat,

diantaranya penjumlahan(+), pengurangan(−), perkalian(× atau ·) dan pembagian(:

atau /). Untuk sebarang dua bilangan bulat berlaku jumlah, selisih dan hasil

kalinya masing-masing merupakan bilangan bulat, tetapi pembagian bilangan

yang satu dengan yang lain belum tentu merupakan bilangan bulat.

Definisi 2.2.1. Diberikan bilangan bulat m dan n (n = 0). Bilangan m dikatakan

habis dibagi oleh n atau n membagi m, ditulis n|m, jika terdapat bilangan bulat

k dengan sifat m = kn. Jika m habis dibagi oleh n, maka m disebut kelipatan

dari n dan n disebut pembagi atau faktor dari m.

Jika m tidak habis dibagi oleh n, dituliskan n | m. Karena 0 = 0.n,

diperoleh bahwa n|0 untuk setiap bilangan bulat n. Sebaliknya, 0 | m untuk

11

setiap bilangan bulat tak nol m, sebab m = 0 = k.0 untuk setiap bilangan bulat

k. Dari definisi keterbagian diperoleh beberapa sifat dasar sebagai berikut.

Teorema 2.2.2. Diberikan bilangan bulat x, y dan z.

a. x|x;

b. Jika x|y dan y|z, maka x|z;

c. Jika x|y dan y = 0, maka |x| ≤ |y|;

d. Jika x|y dan x|z, maka x|αy + βz untuk setiap bilangan bulat α dan β;

e. Jika x|y dan x|y ± z, maka x|z;

f. Jika x|y dan y|x, maka |x| = |y|;

g. Jika x|y dan y = 0, makay

x|y;

h. Untuk z = 0 berlaku x|y jika dan hanya jika xz|yz.

Diperhatikan bahwa untuk sebarang bilangan bulat tak nol n, faktor

positif dari n ada sebanyak ganjil jika dan hanya jika n merupakan kuadrat

sempurna, yaitu n = m2 untuk suatu bilangan bulat m. (Jika suatu bilangan

bulat tidak habis dibagi oleh sebarang bilangan kuadrat, maka bilangan terse-

but disebut square free.) Hal ini dikarenakan jika n bukan kuadrat sempurna,

maka semua faktor positif dari n dapat dinyatakan ke dalam pasangan-pasangan

berbentuk (x, nx).

Contoh 2.2.3. Tentukan semua bilangan bulat n sehinggan+ 20

n− 13merupakan

bilangan bulat.

Penyelesaian. Diperhatikan bahwa

n+ 20

n− 13=

n− 13 + 33

n− 13= 1 +

33

n− 13.

Jikan+ 20

n− 13bulat, maka

33

n− 13bulat. Artinya n − 13|33 atau n − 13 faktor

dari 33. Karena faktor dari 33 adalah −33,−11,−3,−1, 1, 3, 11 dan 33, maka

diperoleh nilai n yang mungkin adalah −20, 2, 10, 12, 14, 16, 24 atau 46. Dapat

12

dicek bahwa semua nilai n tersebut memenuhi kondisi yang diberikan. Jadi, nilai

n yang memenuhi adalah −20, 2, 10, 12, 14, 16, 24 dan 46. �

Contoh 2.2.4. Tentukan semua pasangan bilangan bulat positif (m,n) dengan

sifat2

m+

3

n= 1.

Penyelesaian. Misalkan bilangan bulat positif n dan m memenuhi2

m+

3

n= 1,

maka berlaku

2n+ 3m = mn

⇔ (m− 2)(n− 3) = 6.

Diperoleh bahwa m− 2 dan n− 3 merupakan faktor dari 6. Karena m bilangan

bulat positif, maka m − 2 > −2. Diperoleh nilai m − 2 yang mungkin adalah

1, 2, 3 atau 6, sehingga nilai m yang mungkin adalah 3, 4, 5 atau 8. Akibatnya

diperoleh pasangan (m,n) yang memenuhi adalah (3, 9), (4, 6), (5, 5) dan (8, 4). �

Contoh 2.2.5. Pada suatu ruangan terdapat 20 kotak kosong, bernomor 1 sam-

pai 20. Sebanyak 20 anak secara bergiliran melakukan ekperimen terhadap kotak-

kotak tersebut. Anak pertama memasukkan satu bola ke masing-masing 20 kotak

tersebut. Anak kedua mengambil bola yang ada pada kotak bernomor 2, 4, . . . , 20.

Anak ketiga melakukan eksperimen terhadap kotak-kotak bernomor 3, 6, . . . , 18:

jika pada kotak tidak terdapat bola, maka dia memasukkkan satu bola ke kotak

tersebut dan jika pada kotak terdapat bola, maka dia mengambil bola pada kotak

tersebut. Anak ke i melakukan eksperimen terhadap kotak-kotak bernomor keli-

patan i: jika pada kotak tidak terdapat bola, maka dia memasukkkan satu bola

ke kotak tersebut dan jika pada kotak terdapat bola, maka dia mengambil bola

pada kotak tersebut. Tentukan banyak kotak yang berisi bola setelah semua anak

menyelesaikan eksperimennya?

Penyelesaian. Diperhatikan bahwa anak ke i melakukan eksperimen terhadap

kotak bernomor j jika dan hanya jika i|j. Berdasarkan sifat g. pada Teorema

2.2.2, hal ini terjadi jika dan hanya jika anak kej

imelakukan eksperimen ter-

hadap kotak tersebut. Akibatnya, hanya kotak bernomor 1, 4, 9 dan 16 yang

13

dikenai eksperimen sebanyak bilangan ganjil, sehingga hanya kotak-kotak terse-

but yang berisi bola setelah semua anak menyelesaikan eksperimennya. Jadi,

jawabannya adalah 4. �

Berikut diberikan suatu karakteristik terkait keterbagian dari hasil kali

bilangan bulat berurutan.

Teorema 2.2.6. Hasil kali n ≥ 1 bilangan bulat berurutan selalu habis dibagi

oleh n!. (n! = 1× 2× . . .× n)

Bukti. Pertama-tama, akan ditunjukkan perkalian n bilangan bulat positif beru-

rutan habis dibagi oleh n!. Akan digunakan induksi matematika untuk membuk-

tikannya.

Basis induksi. Untuk n = 1, cukup jelas bahwa perkalian 1 bilangan bulat

positif pasti habis dibagi oleh 1. Jadi, pernyataan benar untuk kasus n = 1.

Langkah induksi. Diasumsikan pernyataan benar untuk n = k, yaitu perkalian

k bilangan bulat positif berurutan habis dibagi oleh k!. Akan ditunjukkan perny-

ataan benar untuk kasus n = k + 1, yaitu perkalian k + 1 bilangan bulat positif

berurutan habis dibagi oleh (k+1)!. Misalkan k+1 bilangan berurutan dimaksud

adalah m,m+1,m+2, . . . ,m+k untuk suatu bilangan bulat positif m. Akan di-

tunjukkan dengan induksi matematika bahwa untuk setiap bilangan bulat positif

m berlaku m(m+ 1)(m+ 2) . . . (m+ k) habis dibagi oleh (k + 1)! .

Basis induksi. Untuk m = 1, diperoleh 1(1 + 1)(1 + 2) . . . (1 + k) =

(k + 1)! = 1.(k + 1)!. Artinya, 1(1 + 1)(1 + 2) . . . (1 + k) habis dibagi oleh

(k + 1)!. Jadi, pernyataan benar untuk m = 1.

Langkah induksi. Diasumsikan pernyataan benar untuk m = p, yaitu

p(p + 1) (p + 2) . . . (p + k) habis dibagi oleh (k + 1)!. Akan ditunjukkan

pernyataan benar untuk kasus m = p+1, yaitu (p+1)(p+2)(p+3) . . . (p+

k + 1) habis dibagi oleh (k + 1)!. Diperhatikan bahwa

(p+ 1)(p+ 2)(p+ 3) . . . (p+ k + 1) = (p+ 1)(p+ 2) . . . (p+ k)p

+(p+ 1)(p+ 2) . . . (p+ k)(k + 1)

= p(p+ 1)(p+ 2) . . . (p+ k)

+(k + 1)(p+ 1)(p+ 2)(p+ 3) . . . (p+ k).

14

Berdasarkan asumsi induksi, diperoleh p(p+1)(p+2) . . . (p+k) habis dibagi

(k + 1)!. Karena (p + 1)(p + 2)(p + 3) . . . (p + k) merupakan perkalian k

bilangan bulat positif berurutan, maka (p+1)(p+2)(p+3) . . . (p+k) habis

dibagi k!, sehingga diperoleh (k + 1) (p+ 1)(p + 2)(p + 3) . . . (p + k) habis

dibagi (k+1)!. Jadi, (p+1)(p+2)(p+3) . . . (p+k+1) habis dibagi (k+1)!.

Terbukti pernyataan benar untuk m = p+ 1.

Jadi, terbukti bahwa perkalian n bilangan bulat positif berurutan habis dibagi

oleh n!.

Selanjutnya, jika diantara n bilangan bulat berurutan terdapat 0, maka hasil

kalinya sama dengan 0, sehingga pasti habis dibagi oleh n!. Untuk kasus, jika

n bilangan berurutan tersebut semua merupakan bilangan negatif, dapat dibuk-

tikan dengan cara yang sama seperti bagian pertama dengan mengalikan hasil

kalinya dengan (−1)n. �

Contoh 2.2.7. Tunjukkan bahwa n6−n2 selalu habis dibagi oleh 60 untuk semua

bilangan bulat positif n.

Penyelesaian. Diberikan sebarang bilangan bulat positif n. Diperhatikan bahwa

n6 − n2 = n2(n4 − 1) = (n− 1)n2(n+ 1)(n2 + 1)

= (n− 1)n2(n+ 1)(n2 − 4) + 5(n− 1)n2(n+ 1)

= (n− 2)(n− 1)n(n+ 1)(n+ 2)n+ 5(n− 1)(n2 − 2n)(n+ 1)

+10(n− 1)n(n+ 1)

= (n− 2)(n− 1)n(n+ 1)(n+ 2)n+ 5(n− 2)(n− 1)n(n+ 1)

+10(n− 1)n(n+ 1).

Diperhatikan bahwa 5!|(n−2)(n−1)n(n+1)(n+2), 4!|(n−2)(n−1)n(n+1) dan

3!|(n−1)n(n+1). Karena 5! = 120, 4! = 24 dan 3! = 6, maka diperoleh 120|(n−2)

(n− 1)n(n+ 1)(n+ 2)n, 120|5(n− 2)(n− 1)n(n+ 1) dan 60|10(n− 1)n(n+ 1).

Karena 60|120, maka 60|(n−2)(n−1)n(n+1)(n+2)n, 60|5(n−2)(n−1)n(n+1)

dan 60|10(n− 1)n(n+ 1), sehingga diperoleh 60|n6 − n2. �

Berdasarkan konsep keterbagian terkait bilangan 2, himpunan bilangan

15

bulat yang dinotasikan Z dapat dipartisi menjadi dua himpunan bagian, him-

punan bilangan ganjil dan himpunan bilangan genap:

{±1,±3,±5, . . .} dan {0,±2,±4, . . .}.

Beberapa konsep dasar yang dimiliki oleh bilangan ganjil dan genap sebagai

berikut:

a. Bilangan ganjil berbentuk 2k + 1 untuk suatu bilangan bulat k;

b. Bilangan genap berbentuk 2k untuk suatu bilangan bulay k;

c. Jumlahan dua bilangan ganjil adalah bilangan genap;

d. Jumlahan dua bilangan genap adalah bilangan genap;

e. Jumlahan bilangan ganjil dan bilangan genap adalah bilangan genap;

f. Perkalian dua bilangan ganjil adalah bilangan ganjil;

g. Perkalian dua bilangan bulat merupakan bilangan genap jika dan hanya

jika salah satunya merupakan bilangan genap.

Konsep ini sangat bermanfaat dalam menyelesaikan beberapa masalah teori bi-

langan.

Contoh 2.2.8. Diberikan bilangan bulat positif n ≥ 1. Tunjukkan bahwa

a. 2n dapat dinyatakan sebagai jumlahan dua bilangan ganjil berurutan.

b. 3n dapat dinyatakan sebagai jumlahan tiga bilangan bulat berurutan.

Penyelesaian. Untuk a., persamaan 2n = (2k−1)+(2k+1) memberikan penye-

lesaian k = 2n−2, sehingga diperoleh 2n = (2n−1 − 1) + (2n−1 + 1).

Untuk b., persamaan 3n = (s−1)+s+(s+1) memberikan penyelesaian s = 3n−2,

sehingga diperoleh 3n = (3n−1 − 1) + 3n−1 + (3n−1 + 1). �

Contoh 2.2.9. Diberikan bilangan ganjil a, b dan c. Tunjukkan bahwa akar-akar

persamaan kuadrat ax2 + bx+ c = 0 bukan bilangan bulat.

16

Penyelesaian. Diandaikan bilangan bulat n merupakan akar persamaan ax2 +

bx+c = 0. Diperoleh an2+bn+c = 0. Akan ditinjau dua kasus, yaitu n bilangan

genap dan n bilangan ganjil.

Kasus n bilangan genap. Karena a dan b bilangan ganjil, maka an2 dan bn meru-

pakan bilangan ganjil, sehingga diperoleh an2 + bn bilangan genap. Karena c bi-

langan ganjil, maka an2+bn+c bilangan ganjil, sehingga diperoleh an2+bn+c = 0

(0 genap), suatu kontradiksi.

Kasus n bilangan ganjil. Karena a dan b bilangan ganjil, maka an2 dan bn meru-

pakan bilangan genap, sehingga diperoleh an2 + bn bilangan genap. Karena c bi-

langan ganjil, maka an2+bn+c bilangan ganjil, sehingga diperoleh an2+bn+c = 0

(0 genap), suatu kontradiksi.

Jadi, akar-akar persamaan kuadrat ax2 + bx+ c = 0 bukan bilangan bulat. �

Contoh 2.2.10. Diberikan k bilangan genap. Tunjukkan bahwa tidak ada bilan-

gan ganjil n1, n2, . . . , nk dengan sifat

1 =1

n1

+1

n2

+ . . .+1

nk

.

Penyelesaian. Diandaikan terdapat bilangan ganjil n1, n2, . . . , nk dengan sifat

1 =1

n1

+1

n2

+ . . .+1

nk

.

Dengan menyamakan penyebut pada ruas kanan, diperoleh n1n2 . . . nk = s1 +

s2 + . . . + sk dengan s1, s2, . . . , sk bilangan ganjil. Diperhatikan bahwa ruas kiri

merupakan bilangan ganjil, sedangkan ruas kanan merupakan bilangan genap,

suatu kontradiksi. �

2.3 Algoritma Pembagian

Berikut diberikan salah satu konsep yang disebut Algoritma Pembagian

yang memiliki peranan penting dalam teori bilangan.

Teorema 2.3.1 (Algoritma Pembagian). Untuk setiap bilangan bulat positif a

dan b terdapat dengan tunggal pasangan bilangan bulat non-negatif (q, r) dengan

sifat b = aq+ r dan r < a. Lebih lanjut, q disebut hasil bagi dan r disebut sisa

ketika b dibagi oleh a.

17

Bukti. Diberikan sebarang bilangan bulat positif a dan b. Pertama-tama, di-

tunjukkan eksistensi dari pasangan (q, r). Diperhatikan bahwa ada 3 kasus yang

mungkin yaitu a < b, a = b atau a > b.

1. Kasus a > b. Dipilih q = 0 dan r = b < a, diperoleh (q, r) = (0, b)

memenuhi kondisi b = aq + r dan r < a.

2. Kasus a = b. Dipilih q = 1 dan r = 0 < a, diperoleh (q, r) = (1, 0)

memenuhi kondisi b = aq + r dan r < a.

3. Kasus a < b. Diperhatikan bahwa terdapat bilangan bulat positif n se-

hingga na > b. Dipilih q bilangan bulat positif terkecil dengan sifat (q +

1)a > b, maka berlaku qa ≤ b. Dipilih r = b−aq. Diperoleh (q, r) memenuhi

kondisi b = aq + r dan 0 ≤ r < a.

Selanjutnya, akan ditunjukkan ketunggalan pasangan (q, r) tersebut. Diandaikan

(q′, r′) memenuhi kondisi b = aq′+ r′ dan 0 ≤ r′ < a. Diperoleh aq+ r = aq′+ r′,

ekuivalen dengan a(q− q′) = r′ − r, yang berarti a|r′ − r. Akibatnya |r′ − r| ≥ a

atau |r′ − r| = 0. Karena 0 ≤ r, r′ ≤ a, maka |r′ − r| < a. Diperoleh |r′ − r| = 0,

artinya r′ = r, sehingga berakibat q = q′. �

Contoh 2.3.2. Diketahui bilangan 1059, 1417 dan 2312 memiliki sisa yang sama

ketika dibagi oleh d > 1. Tentukan nilai d.

Penyelesaian. Misalkan sisanya adalah r. Berdasarkan Algoritma Pembagian,

diperoleh

1059 = q1d+ r

1417 = q2d+ r

2312 = q3d+ r,

untuk suatu bilangan bulat q1, q2 dan q3. Diperoleh

(q2 − q1)d = 1417− 1059 = 358 = 2.179

(q3 − q1)d = 2312− 1059 = 1253 = 7.179

(q3 − q2)d = 2312− 1417 = 895 = 5.179,

18

yang berarti d merupakan faktor dari 2.179, 7.179 dan 5.179. Karena d > 1, maka

diperoleh d = 179. �

Contoh 2.3.3. Diberikan bilangan bulat positif n. Tunjukkan bahwa 32n+1 habis

dibagi oleh 2 tetapi tidak habis dibagi oleh 4.

Penyelesaian. Diperhatikan bahwa 32nmerupakan bilangan ganjil, sehingga

diperoleh 32n+1 bilangan genap, yang berarti habis dibagi oleh 2. Karena 32

n=

(32)2n−1

= 92n−1

= (8 + 1)2n−1

, maka berdasarkan teorema Binomial Newton:

(x+ y)m = xm +

(m

1

)xm−1y +

(m

2

)xm−2y2 + . . .+

(m

m− 1

)xym−1 + ym,

dengan mengambil m = 2n−1, x = 8 dan y = 1 diperoleh

(8 + 1)2n−1

= 82n−1

+

(2n−1

1

)82

n−1−1 +

(2n−1

2

)8m−2 + . . .+

(2n−1

2n−1 − 1

)8 + 1.

Akibatnya 32n+ 1 = (8 + 1)2

n−1+ 1 = (8K + 1) + 1 = 4(2K) + 2 untuk suatu

bilangan bulat positif K. Jadi, 32n+1 habis dibagi oleh2 tetapi tidak habis dibagi

oleh 4. �

Algoritma Pembagian tidak hanya berlaku untuk bilangan bulat positif

saja, tetapi dapat diperluas untuk bilangan bulat. Bukti diserahkan sebagai

latihan.

Teorema 2.3.4. Untuk setiap bilangan bulat a dan b (a = 0), terdapat dengan

tunggal pasangan bilangan bulat non-negatif (q, r) dengan sifat b = aq + r dan

0 ≤ r < |a|.

2.4 Bilangan Prima

Pada bagian ini dijelaskan mengenai konsep bilangan prima dan bilangan

komposit.

Definisi 2.4.1. Bilangan bulat p > 1 dikatakan prima jika untuk setiap bilangan

bulat d dengan d > 1, d = p berlaku d | p. Bilangan bulat n > 1 yang tidak prima

dikatakan komposit.

19

Diperhatikan bahwa setiap bilangan bulat n > 1 mempunyai setidaknya

satu faktor prima. Untuk n prima, faktor primanya adalah n sendiri. Untuk

n bukan prima, misalkan a adalah faktor positif terkecil dari n. Diperoleh a

merupakan bilangan prima, sebab jika a bukan prima, maka a = a1a2 untuk

suatu 1 ≤ a1, a2 < a dan a1|n, kontradiksi dengan fakta bahwa a faktor positif

terkecil dari n. Berikut diberikan suatu sifat yang bermanfaat dalam menentukan

suatu bilangan adalah prima atau tidak.

Teorema 2.4.2. Diberikan bilangan bulat n > 1. Jika n komposit, maka n

memiliki faktor prima yang kurang dari atau sama dengan√n.

Bukti. Diketahui n komposit. Misalkan n = ab untuk suatu a, b dengan

1 < a ≤ b dan a faktor positif terkecil dari n. Diperoleh n = ab ≥ a2, se-

hingga diperoleh a ≤√n. �

Diperhatikan bahwa 2 merupakan bilangan prima genap dan semua bi-

langan genap lebih dari dua merupakan bilangan komposit. Bilangan prima yang

lain merupakan bilangan ganjil. Bilangan-bilangan prima yang kurang dari 50

adalah 2, 3, 5, 7, 11, 13, 17, 19, 23, 29, 31, 37, 41, 43, 47.

Contoh 2.4.3. Diketahui p dan q bilangan prima yang memenuhi p+ q = 2013

dan p > q. Tentukan nilai dari p− q.

Penyelesaian. Diperhatikan bahwa salah satu diantara p dan q bilangan genap

sebab jika keduanya ganjil atau keduanya genap, maka p + q genap, kontradiksi

dengan fakta bahwa p+ q = 2013 bilangan ganjil. Karena bilangan prima genap

hanya 2 dan p > q, maka diperoleh q = 2, sehingga didapat p = 2011. Diper-

hatikan bahwa 2011 tidak habis dibagi oleh sebarang bilangan prima yang kurang

dari√2011, yaitu 2,3,5,7,11,13,17,19,23,29,31,37,41 atau 43. Jadi, 2011 meru-

pakan bilangan prima. Akibatnya diperoleh p− q = 2011− 2 = 2009. �

Contoh 2.4.4. Tentukan semua bilangan bulat positif n dengan sifat 3n−4, 4n−5

dan 5n− 3 merupakan bilangan prima.

Penyelesaian. Diperhatikan bahwa jumlah ketiga bilangan tersebut adalah

bilangan genap, maka setidaknya salah satu diantaranya merupakan bilangan

20

genap. Satu-satunya bilangan prima genap adalah 2. Dari ketiga bilangan terse-

but, hanya 3n−4 dan 5n−3 yang mungkin bernilai genap. Untuk kasus 3n−4 = 2,

diperoleh n = 2. Untuk kasus 5n − 3 = 2, diperoleh n = 1. Dapat dicek bahwa

hanya n = 2 yang memenuhi kondisi ketiga bilangan tersebut merupakan bilan-

gan prima. �

Contoh 2.4.5. Tunjukkan bahwa n4 + 4 merupakan bilangan prima jika dan

hanya jika n = 1.

Penyelesaian. Diperhatikan bahwa

n4 + 4 = n4 + 4n2 + 4− 4n2 = (n2 + 2)2 − (2n)2

= (n2 − 2n+ 2)(n2 + 2n+ 2) = ((n− 1)2 + 1)((n+ 1)2 + 1).

Diperhatikan bahwa untuk n > 1 berlaku (n− 1)2 + 1 > 1 dan (n+ 1)2 + 1 > 1.

Akibatnya, n4 + 4 bukan bilangan prima untuk n > 1. �

Contoh 2.4.6. Carilah 20 bilangan bilangan bulat berurutan yang masing-masing

merupakan bilangan komposit.

Penyelesaian. Diperhatikan 20 bilangan berurutan berikut 21!+2, 21!+3, . . . , 21!+

21. Untuk setiap i = 2, . . . , 21, 21! + i merupakan bilangan komposit sebab

i|(20! + i). �

Lebih dari 2000 tahun yang lalu, Euclid telah menunjukkan bahwa ada

tak hingga banyak bilangan bulat positif yang merupakan bilangan prima.

Teorema 2.4.7. Ada tak hingga banyaknya bilangan prima.

Bukti. Diandaikan bilangan prima hanya berhingga banyak, katakan p1 < p2 <

. . . < pm. Diperhatikan bilangan P = p1p2 . . . pm + 1. Jika P prima, maka

P > pm, kontradiksi dengan fakta bahwa pm bilangan prima terbesar. Akibat-

nya P haruslah komposit. Artinya P memiliki faktor prima, katakan p > 1.

Diperhatikan bahwa p = pk untuk suatu k ∈ {1, 2, . . . ,m}. Diperoleh bahwa

pk|p1p2 . . . pk . . . pm + 1. Artinya pk|1, suatu kontradiksi. Jadi, ada tak hingga

21

banyaknya bilangan prima. �

Walaupun telah diketahui bahwa banyaknya bilangan prima ada tak

berhingga, namun sampai saat ini masih belum ditemukan suatu formula untuk

menentukan semua bilangan prima yang ada.

Soal Latihan

1. Tunjukkan bahwa 15+25+ . . .+995+1005 habis dibagi 10100, namun tidak

habis dibagi 3.

2. Diketahui p dan p+ 2 adalah bilangan prima yang lebih besar dari 3. Ten-

tukan sisa dari p ketika dibagi oleh 6.

3. Tentukan bilangan bulat positif n terbesar sehingga n+ 10 habis membagi

n3 + 100.

4. Tunjukkan bahwa untuk setiap bilangan bulat positif n berlaku n5−5n3+4n

habis dibagi oleh 120.

5. Diketahui x, y dan z adalah bilangan prima yang memenuhi persamaan

34x− 51y = 2012z.

Tentukan nilai dari x+ y + z.

6. Tentukan semua bilangan bulat positif n sehingga n4 + 4n merupakan bi-

langan prima.

7. Diketahui m dan n adalah bilangan bulat yang memenuhi m2 + 3m2n2 =

30n2 + 517. Tentukan nilai dari 2m2n2.

8. Tunjuukan bahwa jika |ab| = 1, maka a4+4b4 merupakan bilangan komposit.

9. Diberikan polinomial p(x) = xn+a1xn−1+. . .+an−1x+an dengan a1, a2, . . . , an

bilangan bulat. Jika p(0) dan p(1) keduanya bilangan ganjil, tunjukkan

bahwa p(x) tidak memiliki akar bulat.

22

10. Tentukan semua bilangan positif p dengan sifat p, p+8 dan p+16 merupakan

bilangan prima.

11. Tentukan semua bilangan bulat n yang memenuhi3n2 + 4n+ 5

2n+ 1bilangan

bulat.

12. (a) Tunjukkan bahwa jika bilangan bulat a dan b bersisa 1 ketika dibagi

oleh 4, maka ab bersisa 1 ketika dibagi oleh 4.

(b) Tunjukkan ada tak hingga banyaknya bilangan prima berbentuk 4k−1.

13. Tentukan semua pasangan bilangan prima berbeda (p, q) sehingga p2+7pq+

q2 merupakan bilangan kuadrat sempurna.

23

BAB III

FAKTORISASI PRIMA

3.1 Pendahuluan

Sebagai kelanjutan dari konsep bilangan prima dan keterbagian, pada bagian

ini dibahas mengenai faktorisasi prima pada bilangan bulat dan aplikasinya un-

tuk menentukan banyak faktor dan jumlah faktor suatu bilangan bulat. Materi

ini disampaikan pada Minggu ke-5 dan 6.

Dengan mempelajari bab ini, diharapkan:

1. Mahasiswa bisa melakukan faktorisasi prima sebarang bilangan bulat.

2. Mahasiswa bisa menentukan banyak faktor bilangan bulat

3. Mahasiswa bisa menentukan jumlah faktor-faktor bilangan bulat

4. Mahasiswa bisa menerapkan sifat-sifat faktorisasi prima masalah bilangan

bulat

3.2 Teorema Fundamental Aritmatik

Salah satu sifat dasar dari teori bilangan terkait dengan faktor prima diberikan

sebagai berikut.

Teorema 3.2.1 (Teorema Fundamental Aritmatik). Setiap bilangan bulat n > 1

dapat dinyatakan sebagai perkalian bilangan-bilangan prima secara tunggal.

Bukti. Diberikan bilangan bulat n > 1. Pertama-tama, akan ditunjukkan eksis-

tensinya. Misalkan p1 faktor prima dari n. Jika p1 = n, maka n = p1 merupakan

faktorisasi prima dari n. Jika p1 < n, maka n = p1r1 dengan r1 > 1. Jika r1

prima, maka n = p1p2 dengan p2 = r1 merupakan faktorisasi yang dimaksud. Jika

r1 komposit, maka r1 = p2r2 dengan p2 prima dan r2 > 1, sehingga n = p1p2r2.

Jika r2 prima, maka n = p1p2p3 dengan p3 = r3 merupakan faktorisasi yang di-

maksud. Jika r2 komposit, maka secara rekursif diperoleh barisan bilangan bulat

24

r1 > r2 > . . . ≥ 1. Setelah sejumlah langkah, diperoleh pk+1 = 1, sehingga dida-

pat n = p1p2 . . . pn merupakan faktorisasi yang dimaksud.

Selanjutnya, akan ditunjukkan ketunggalannya. Diasumsikan n memiliki dua

faktorisasi prima berbeda, yaitu:

n = p1p2 . . . pk = q1q2 . . . qh

dimana p1, p2, . . . , pk, q1, q2, . . . , qh bilangan prima dengan p1 ≤ p2 ≤ . . . ≤ pk

dan q1 ≤ q2 ≤ . . . ≤ qh sehingga k-tupel (p1, p2, . . .) tidak sama dengan h-tupel

(q1, q2, . . . , qh. Jelas bahwa k, h ≥ 2. Misalkan n merupakan bilangan terkecil

yang memiliki dua faktorisasi prima. Akan ditunjukkan terjadi suatu kontradiksi

dengan menemukan bilangan yang lebih kecil dari n yang memiliki dua faktorisasi

prima.

Diperhatikan bahwa pi = qj untuk setiap i = 1, 2, . . . , k, j = 1, 2, . . . , h sebab

jika ada yang sama, misalkan pk = qh = p, maka n′ = n/p = p1p2 . . . pk−1 =

q1q2 . . . qh−1 dan 1 < n′ < n, kontradiksi dengan fakta bahwa n bilangan terkecil

yang memiliki dua faktorisasi prima berbeda. Tanpa mengurangi keumuman,

misalkan p1 ≤ q1. Berdasarkan Algoritma Pembagian diperoleh:

q1 = p1c1 + r1

q2 = p1c2 + r2...

qh = p1ch + rh

dengan 1 ≤ ri < p1, i = 1, . . . , h. Diperoleh bahwa

n = q1q2 . . . qh = (p1c1 + r1)(p1c2 + r2) . . . (p1ch + rh)

Dengan menjabarkan bentuk pada ruas kanan persamaan tersebut diperoleh

n = mp1 + r1r2 . . . rh untuk suatu bilangan bulat m. Dengan mengambil n′ =

r1r2 . . . rh, maka diperoleh n = p1p2 . . . pk = p1m + n′. Diperoleh bahwa p1|n′,

yang artinya n′ = p1s untuk suatu bilangan bulat s. Berdasarkan pembuktian ek-

sistensi faktorisasi prima diperoleh s dapat dituliskan sebagai perkalian bilangan-

bilangan prima, katakan s = s1s2 . . . si dengan s1, s2, . . . , si bilangan prima.

Di lain pihak, dengan menggunakan faktorisasi r1, r2 . . . , rh sebagai perkalian

bilangan-bilangan prima, diperoleh n′ = t1t2 . . . tj dengan t1, t2, . . . , tj bilangan

25

prima. Diperhatikan bahwa tu < p1 untuk setiap u = 1, 2 . . . , j, sehingga fak-

torisasi n′ = t1t2 . . . tj berbeda dengan n′ = p1s1s2 . . . si. Akan tetapi n′ < n,

kontradiksi dengan fakta bahwa n bilangan terkecil yang memiliki dua faktorisasi

prima berbeda. �

Berdasarkan Teorema 3.2.1, diperoleh bahwa setiap bilangan bulat n >

1 dapat dituliskan secara tunggal dalam bentuk

n = pα11 pα2

2 . . . pαkk

dengan p1, p2, . . . , pk bilangan prima berbeda dan α1, α2, . . . , αk. Representasi ini

dinamakan faktorisasi prima (faktorisasi kanonik ) dari n.

Contoh 3.2.2. Tunjukkan bahwa m5+3m4− 5m3− 15m2+4m+12 = 33 untuk

setiap bilangan bulat positif m dan n.

Penyelesaian. Diperhatikan bahwa

m5 + 3m4 − 5m3 − 15m2 + 4m+ 12 = (m− 2)(m− 1)(m+ 1)(m+ 2)(m+ 3).

Di lain pihak, 33 dapat dinyatakan sebagai perkalian maksimal sebanyak empat

bilangan bulat berbeda, yaitu 33 = (−11)(−3)1(−1). Berdasarkan Teorema Fun-

damental Aritmatik, m5 + 3m4 − 5m3 − 15m2 + 4m + 12 = 33 sebab 33 dapat

dinyatakan sebagai perkalian maksimal sebanyak empat bilangan bulat berbeda,

sedangkan m5+3m4−5m3−15m2+4m+12 dapat dinyatakan sebagai perkalian

lima bilangan bulat berbeda. �

Contoh 3.2.3. Tentukan semua bilangan bulat positif n sehingga 28 + 211 + 2n

merupakan bilangan kuadrat sempurna.

Penyelesaian. Misalkan 28 + 211 + 2n bilangan kuadrat sempurna. Artinya,

k2 = 28+211+2n = 2304+2n = 482+2n. Diperoleh 2n = k2−482 = (k−48)(k+48).

Berdasarkan ketunggalan dari faktorisasi prima, diperoleh k − 48 = 2s dan

k+48 = 2t untuk suatu bilangan bulat positif s, t dengan s+t = n. Diperhatikan

bahwa 2t−2s = 96 = 3.25 atau 2s(2t−s−1) = 3.25. Berdasarkan ketunggalan dari

faktorisasi prima, diperoleh s = 5, s− t = 2, sehingga diperoleh n = s+ t = 12. �

26

Dapat dicek bahwa faktorisasi prima dari perkalian dua bilangan bulat

sama dengan perkalian dari faktorisasi prima dua bilangan tersebut. Hal ini

memberikan suatu karakteristik lain terkait bilangan prima.

Teorema 3.2.4. Diberikan bilangan bulat a dan b. Jika bilangan prima p mem-

bagi ab, maka p membagi a atau p membagi b.

Bukti. Karena p|ab, maka p harus muncul pada faktorisasi prima dari ab. Karena

faktorisasi prima dari a, b dan ab tunggal dan faktorisasi prima dari ab merupakan

perkalian faktorisasi prima dari a dan b, maka p harus muncul setidaknya pada

salah satu faktorisasi prima dari a atau b, yang berarti p|a atau p|b. �

Definisi 3.2.5. Diberikan bilangan bulat n > 1 dan bilangan prima p. Bilangan

pk dikatakan membagi penuh n, ditulis pk∥n, jika k adalah bilangan bulat positif

terbesar sehingga pk|n.

Contoh 3.2.6. Tentukan faktor terbesar dari 1001001001 yang kurang dari 10000.

Penyelesaian. Diperhatikan bahwa

1001001001 = 1001 · 106 + 1001 = 1001(106 + 1) = 7 · 11 · 13 · (106 + 1).

Karena x6+1 = (x2)3+1 = (x2+1)(x4−x2+1), maka diperoleh 106+1 = 101·9901.Jadi, 1001001001 = 7 · 11 · 13 · 101 · 9901. Dapat dicek bahwa faktor terbesar dari

1001001001 yang kurang dari 10000 adalah 9901. �

Contoh 3.2.7. Tentukan bilangan bulat positif n yang memenuhi 2n∥31024 − 1.

Penyelesaian. Diperhatikan bahwa 210 = 1024 dan x2 − y2 = (x + y)(x − y),

sehingga diperoleh

3210

= (329

+ 1)(329 − 1) = (32

9

+ 1)(328

+ 1)(328 − 1)

= . . . = (329

+ 1)(328

+ 1)(327

+ 1) . . . (321

+ 1)(320

+ 1)(3− 1).

Berdasarkan Contoh 2.3.3, 2∥32k untuk setiap bilangan bulat positif k. Jadi,

jawabannya adalah 9 + 2 + 1 = 12. �

27

Berdasarkan Teorema 3.2.1, diperoleh bahwa setiap bilangan bulat diban-

gun oleh bilangan-bilangan prima. Karena pentingnya konsep bilangan prima,

banyak peneliti telah memcoba menemukan rumus eksplisit dari bilangan-bilangan

prima. Namun, sejauh ini usaha tersebut belum berhasil. Salah satu hasil yang

diperoleh Goldbach dalam penelitiannya terkait bilangan prima diberikan sebagai

berikut.

Teorema 3.2.8. Untuk setiap bilangan bulat n tidak ada polinomial p(x) dengan

koefisien bulat dengan sifat p(n) merupakan bilangan prima untuk setiap bilangan

bilat n ≥ m.

Bukti. Diberikan bilangan bulat m. Diandaikan terdapat polinomial yang

memenuhi kondisi tersebut, katakan

P (x) = akxk + ak−1x

k−1 + . . .+ a1x+ a0

dengan ak, ak−1, . . . , a0 bilangan bulat dan ak = 0.

Diperoleh p(m) = p bilangan prima. Diperhatikan bahwa

p(m+ pi) = ak(m+ pi)k + ak−1(m+ pi)k−1 + . . .+ a1(m+ pi) + a0

dan untuk setiap bilangan bulat positif i berlaku

(m+ pi)j = mj +

(j

i

)mj−1(pi) +

(j

2

)mj−2(pi)2

+ . . .+

(j

j − 1

)m(pi)j−1 + (pi)j

untuk setiap j = 1, 2, . . . , k. Diperoleh bahwa (m + pi)j − mj kelipatan dari p,

sehingga berlaku p(m+pi)−p(m) merupakan kelipatan dari p. Karena p(m) = p,

maka diperoleh p(m+ pi) merupakan kelipatan dari p untuk setiap bilangan bu-

lat positif i. Berdasarkan asumsi diperoleh bahwa p(m+ pi) prima untuk setiap

bilangan bulat positif i. Akibatnya nilai yang mungkin untuk p(m + pi) adalah

0, p atau −p. Diperhatikan bahwa total akar persamaan p(x) = 0, p(x) = p

dan p(x) = −p paling banyak 3k. Akibatnya terdapat tak hingga banyaknya i

dengan sifat m + pi bukan solusi dari ketiga persamaan tersebut. Terjadi su-

atu kontradiksi. Jadi, untuk setiap bilangan bulat n, tidak ada polinomial p(x)

dengan koefisien bulat dengan sifat p(n) merupakan bilangan prima untuk setiap

28

bilangan bilat n ≥ m. �

Walaupun belum ada rumus eksplisit dari bilangan prima, rata-rata

banyaknya bilangan prima diantara bilangan bulat telah diketahui 100 tahun

yang lalu. Fakta ini diberikan oleh Hadamard dan de la Vallee Poussin pada

tahun 1896, yaitu:

limn→∞

π(n)

n/ log n= 1

dengan π(n) merupakan banyaknya bilangan prima yang kurang dari atau sama

dengan n.

3.3 Banyak Faktor

Untuk setiap bilangan bulat positif n, banyaknya faktor positif dari n dinotasikan

dengan τ(n). Jelas bahwa

τ(n) =∑d|n

1.

Teorema 3.3.1. Jika n = pα11 pα2

2 . . . pαkk faktorisasi prima dari n, maka

τ(n) = (α1 + 1)(α2 + 1) . . . (αk + 1).

Bukti. Diperhatikan bahwa berdasarkan faktorisasi prima dari n, setiap fak-

tor positif dari n berbentuk pb11 pb22 . . . pbkk , dengan 0 ≤ bi ≤ αi, i = 1, 2, . . . , k.

Diperoleh banyaknya faktor positif dari n sama dengan banyaknya kemungkinan

nilai dari b1, b2, . . . , bn. Karena untuk setiap i, ada (αi+1) kemungkinan untuk bi,

maka diperoleh banyaknya faktor positif dari n adalah (α1+1)(α2+1) . . . (αk+1).

Teorema 3.3.2. Untuk setiap bilangan bulat positif n berlaku∏d|n

d = nτ(n)2 .

29

Bukti. Diperhatikan bahwa∏d|n

d

2

=∏d|n

d∏d|n

d =∏d|n

d∏d|n

n

d

=∏d|n

(d.n

d

)=∏d|n

n = nτ(n).

Jadi, ∏d|n

d = nτ(n)2 .

Contoh 3.3.3. Tentukan peluang sebarang bilangan dipilih dari faktor positif 1020

merupakan kelipatan 1013.

Penyelesaian. Diperhatikan bahwa setiap faktor positif dari 1020 berbentuk

2a5b dengan 0 ≤ a, b ≤ 20, sedangkan 1013 = 213513. Diperoleh faktor posi-

tif dari faktor positif 1020 yang merupakan kelipatan 1013 berbentuk 2a5b den-

gan 13 ≤ a, b ≤ 20, sehingga didapat banyaknya faktor yang memenuhi kondisi

tersebut ada 8 × 8 = 64. Di lain pihak, banyak faktor positif dari 1020 adalah

21 × 21 = 441. Jadi, peluang sebarang bilangan dipilih dari faktor positif 1020

merupakan kelipatan 1013 adalah64

441. �

Teorema 3.3.4. Untuk setiap bilangan bulat positif n berlaku τ(n) ≤ 2√n.

Bukti. Misalkan d1 < d2 < . . . < dk faktor-faktor positif dari n yang kurang

dari atau sama dengan√n. Diperoleh bahwa faktor lain yang tersisa adalah

n

d1,n

d2, . . . ,

n

dk.

Akibatnya diperoleh τ(n) ≤ 2k ≤ 2dk ≤ 2√n. �

30

3.4 Jumlah Faktor

Untuk setiap bilangan bulat positif n, jumlah semua faktor positif dari n, terma-

suk 1 dan n, dinotasikan dengan σ(n). Jelas bahwa

σ(n) =∑d|n

d.

Teorema 3.4.1. Jika n = pα11 pα2

2 . . . pαkk faktorisasi prima dari n, maka

σ(n) =pα1+11 − 1

p1 − 1. . .

pαk+1k − 1

pk − 1.

Bukti. Diperhatikan bahwa setiap faktor positif dari n berbentuk

pa11 pa22 . . . pakk

dengan 0 ≤ bi ≤ αi, i = 1, 2, . . . , k. Setiap faktor positif dari n muncul tepat

sekali pada penjabaran perkalian

(1 + p1 + . . .+ pα11 ) . . . (1 + pk + . . .+ pαk

k ).

Akibatnya diperoleh

σ(n) = (1 + p1 + . . .+ pα11 ) . . . (1 + pk + . . .+ pαk

k ) =pα1+11 − 1

p1 − 1. . .

pαk+1k − 1

pk − 1.

Contoh 3.4.2. Tentukan jumlah semua faktor positif genap dari 10000.

Penyelesaian. Diperhatikan bahwa setiap faktor positif dari 10000 berbentuk

2a5b dimana a, b bilangan bulat dengan 1 ≤ a ≤ 5 dan 0 ≤ b ≤ 5. Setiap faktor

genap dari 10000 muncul tepat sekali pada penjabaran perkalian

(2 + 22 + 23 + 24 + 25)(1 + 5 + 52 + 53 + 54 + 55).

Akibatnya diperoleh

(2 + 22 + 23 + 24 + 25)(1 + 5 + 52 + 53 + 54 + 55) = 62.56 − 1

5− 1= 242172.

31

Diperhatikan bahwa p prima jika dan hanya jika faktor positif dari p hanya 1

dan p, yang berarti σ(p) = p+ 1. Akibatnya diperoleh karakteristik dari jumlah

faktor terkait bilangan prima.

Teorema 3.4.3. Bilangan bulat positif p merupakan bilangan prima jika dan

hanya jika σ(p) = p+ 1.

Teorema 3.4.4. Untuk setiap bilangan komposit n berlaku σ(n) > n+√n.

Bukti. Diberikan sebarang bilangan komposit n. Karena n komposit, maka

berdasarkan Teorema 2.4.2 terdapat a faktor positif dari n dengan 1 < a ≤√n.

Karena a faktor dari n, maka namerupakan faktor dari n dengan n

a≥ n√

n=

√n.

Diperoleh 1, a, na, n merupakan faktor positif dari n, sehingga didapat σ(n) ≥

1 + a+ na+ n > n+

√n. �

Soal Latihan

1. Tentukan bilangan bulat positif terkecil yang memiliki tepat 12 faktor posi-

tif.

2. Diketahui n = 231319. Tentukan banyaknya faktor positif dari n2 yang

kurang dari n tetapi tidak membagi n.

3. Berapa banyak pembagi genap dan pembagi ganjil dari 56 − 1.

4. Tentukan bilangan bulat positif terkecil n yang mempunyai tepat 2013 fak-

tor positif dan merupakan kelipatan dari 2013.

5. Tentukan semua nilai n sehingga σ(n) merupakan bilangan ganjil.

6. Tunjukkan bahwa untuk setiap bilangan bulat positif n berlaku τ(n) ≤√3n.

7. Diberikan polinomial p(x) dengan koefisien bilangan bulat dan terdapat

bilangan bulat berbeda a, b, c, d dengan sifat p(a) = p(b) = p(c) = p(d) = 5.

Tunjukkan bahwa tidak ada bilangan bulat k dengan sifat f(k) = 8.

32

8. Diberikan n = pa11 pa22 . . . pakk dan m = pb11 pb22 . . . pbkk dengan p1, p2, . . . , pk

bilangan prima berbeda dan a1, a2, . . . , ak, b1, . . . , bk bilangan bulat non-

negatif. Tentukan banyaknya faktor positif dari n yang merupakan faktor

dari m.

9. Bilangan bulat positif n dikatakan sempurna jika n sama dengan jumlahan

semua faktor positif dari n yang kurang dari n. Tunjukkan bahwa

(a) jika n bilangan sempurna, maka∑d|n

1

d= 2.

(b) untuk setiap bilangan bulat positif k dan bilangan prima p, pk bukan

bilangan sempurna.

10. Diberikan bilangan bulat positif k. Tunjukkan bahwa hanya ada berhingga

banyak bilangan bulat positif n dengan sifat σ(n) = n+ k.

11. Diberikan bilangan bulat positif n dengan sifat 24|n+1. Tunjukkan bahwa

24|σ(n).

33

BAB IV

FAKTOR PERSEKUTUAN DAN KELIPATAN

PERSEKUTUAN

4.1 Pendahuluan

Pada bagian ini dibahas konsep mengenai faktor persekutuan terbesar dan

kelipatan persekutuan terkecil bilangan-bilangan bulat. Faktorisasi prima yang

telah dibahas pada Bab 2, pada pertemuan Minggu ke-8 dan 9 memunculkan

konsep faktor persekutuan dan kelipatan persekutuan antara lebih dari satu bi-

langan bulat.

Dengan mempelajari bab ini, diharapkan:

1. Mahasiswa bisa menjelaskan pengertian faktor persekutuan dan faktor perseku-

tuan terbesar.

2. Mahasiswa bisa menentukan FPB dua atau lebih bilangan bulat

3. Mahasiswa bisa menjelaskan pengertian kelipatan persekutuan dan keli-

patan persekutuan terkecil

4. Mahasiswa bisa menentukan kelipatan persekutuan terkecil

5. Mahasiswa bisa menerapkan sifat-sifat FPB dan KPK pada masalah bilan-

gan bulat

4.2 Faktor Persekutuan Terbesar

Untuk setiap bilangan bulat positif k, didefinisikan Dk sebagai himpunan

semua faktor positif dari k. Jelas bahwa Dk merupakan himpunan berhingga.

Definisi 4.2.1. Diberikan bilangan bulat positif m dan n. Anggota terbesar dari

himpunan Dm ∩ Dn disebut faktor persekutuan terbesar (greatest com-

mon divisor) dari m dan n, dinotasikan dengan gcd(m,n). Bilangan m dan n

dikatakan relatif prima jika gcd(m,n) = 1.

34

Beberapa sifat dasar dari faktor persekutuan terbesar diberikan sebagai

berikut.

Teorema 4.2.2. Diberikan bilangan bulat positif m,n dan p.

a. Jika p prima, maka gcd(p,m) = p atau gcd(p,m) = 1.

b. Jika d = gcd(m,n), m = dm′, n = dn′, maka gcd(m′, n′) = 1.

c. Jika d = gcd(m,n), m = d′m”, n = d′n”, gcd(m”, n”) = 1, maka

d′ = d.

d. Jika d′ faktor persekutuan dari m dan n, maka d′ membagi gcd(m,n).

e. Jika px∥m dan py∥n, maka pmin(x,y)∥ gcd(m,n). Lebih lanjut, jika m =

pα11 . . . pαk

k dan n = pβ1

1 . . . pβk

k , αi, βi ≥ 0, i = 1, 2, . . . , k, maka

gcd(m,n) = pmin(α1,β1)1 . . . p

min(αk,βk)k .

f. Jika m = nq + r, maka gcd(m,n) = gcd(n, r).

Contoh 4.2.3. Diberikan d = gcd(7n + 5, 5n + 4), dimana n adalah bilangan

bulat positif.

a. Buktikan bahwa untuk setiap bilangan bulat positif n berlaku d = 1 atau d = 3.

b. Buktikan bahwa d = 3 jika dan hanya jika n = 3k + 1 untuk suatu bilangan

bulat positif k.

Penyelesaian. Diambil sebarang bilangan bulat positif n.

a. Diperhatikan bahwa d|7n+5 dan d|5n+4, maka d|5(7n+5) dan d|7(5n+4).

Akibatnya d|5(7n + 5) − 7(5n + 4) atau d|3. Artinya, d faktor positif dari 3.

Jadi, d = 1 atau d = 3.

b. Diperhatikan bahwa n dapat dinyatakan dalam salah satu bentuk berikut: 3k,

3k + 1 atau 3k + 1, untuk suatu bilangan bulat positif k. Jika n = 3k, maka

7n+ 5 = 21k + 5 = 3(7k + 1) + 2 dan 5n+ 4 = 15k + 4 = 3(5k + 1) + 1. Jika

n = 3k + 1, maka 7n + 5 = 21k + 12 = 3(7k + 4) dan 5n + 4 = 15k + 9 =

3(5k + 3). Jika n = 3k + 2, maka 7n + 5 = 21k + 19 = 3(7k + 6) + 1 dan

35

5n + 4 = 15k + 14 = 3(5k + 4) + 2. Diperoleh 3|7n + 5 dan 3|5n + 4 jika

dan hanya jika n = 3k+1 untuk suatu bilangan bulat positif k. Diperhatikan

bahwa 3|7n+5 dan 3|5n+4 berakibat 3| gcd(7n+5, 5n+4) atau 3|d. Karena

d|3 dan 3|d, maka d = 3. Jadi, d = 3 jika dan hanya jika n = 3k + 1 untuk

suatu bilangan bulat positif k.

Contoh 4.2.4. Tunjukkan bahwa untuk setiap bilangan bulat positif n, pecahan21n+ 4

14n+ 3tidak dapat disederhanakan.

Penyelesaian. Diambil sebarang bilangan bulat positif n. Diperhatikan bahwa

3(14n + 3) − 2(21 + 4) = 1. Akibatnya diperoleh gcd(21n + 4, 14n + 3)|1, yangberarti gcd(21n + 4, 14n + 3) = 1. Dengan kata lain, pecahan

21n+ 4

14n+ 3sudah

dalam bentuk yang sederhana. �

Definisi dari faktor persekutuan terbesar dapat diperluas untuk lebih

dari dua bilangan. Untuk sebarang bilangan bulat positif a1, a2, . . . , an, gcd(a1, a2, . . . , an)

didefinisikan sebagai faktor persekutuan terbesar dari semua bilangan a1, a2, . . . , an.

Berikut beberapa sifat terkait dengan faktor persekutuan terbesar dari beberapa

bilangan bulat.

Teorema 4.2.5. Diberikan a1, a2, . . . , as,m, n, p, d bilangan bulat positif.

a. gcd(gcd(m,n), p) = gcd(m, gcd(n, p)).

b. Jika d|ai, i = 1, 2 . . . , s, maka d| gcd(a1, a2, . . . , as).

c. Jika ai = pα1i1 . . . pαki

k , i = 1, . . . , s, maka

gcd(a1, . . . , as) = pmin(α11,...,α1s)1 . . . p

min(αk1,...,αks)k .

Bilangan a1, a2, . . . , an dikatakan relatif prima jika gcd(a1, a2, . . . , an) =

1. Diperhatikan bahwa gcd(a1, a2, . . . , an) = 1 belum tentu berakibat gcd(ai, aj) =

1 untuk 1 ≤ i < j ≤ n. Jika a1, a2, . . . , an memenuhi gcd(ai, aj) = 1 untuk

1 ≤ i < j ≤ n, maka a1, a2, . . . , an dikatakan sepasang-sepasang relatif prima.

36

Contoh 4.2.6. Tentukan gcd(26 − 22, 36 − 32, 46 − 42, 56 − 52, 66 − 62, 76 − 72).

Penyelesaian. Misalkan d = gcd(26−22, 36−32, 46−42, 56−52, 66−62, 76−72).

Berdasarkan Contoh 2.2.7, untuk setiap bilangan bulat positif n berlaku 60|n6 −n2. Akibatnya diperoleh 60|d. Diperhatikan bahwa karena 26 − 22 = 60, maka

diperoleh d|60. Jadi, d = 60. �

4.3 Algoritma Euclid

Faktorisasi prima dapat membantu menentukan faktor persekutuan terbe-

sar dari bilangan-bilangan bulat positif. Akan tetapi, untuk bilangan yang cukup

besar faktorisasi prima tidak mudah dilakukan. Berikut dijelaskan salah satu

algoritma yang bermanfaat dalam menentukan faktor persekutuan terbesar dari

dua bilangan bulat positif m dan n, yaitu Algoritma Euclid. Algoritma ini

menggunakan algoritma pembagian yang dilakukan berulang-ulang:

m = nq1 + r1, 1 ≤ r1 < n,

n = r1q2 + r2, 1 ≤ r2 < r1,...

rk−2 = rk−1qk + rk, 1 ≤ rk < rk−1,

rk−1 = rkqk+1 + rk+1, rk+1 = 0.

Persamaan-persamaan tersebut ada sebanyak berhingga sebab n > r1 > r2 >

. . . > rk. Berdasarkan sifat f. pada Teorema 4.2.2, diperoleh

gcd(m,n) = gcd(n, r1) = gcd(r1, r2) = . . . = gcd(rk−1, rk) = rk.

Contoh 4.3.1. Jika sebuah bilangan bulat positif kelipatan 305 dipilih secara

acak, dengan setiap kelipatan mempunyai peluang yang sama untuk dipilih, ten-

tukan peluang bilangan tersebut habis dibagi 2013?

37

Penyelesaian. Berdasarkan Algoritma Euclid:

2013 = 6.305 + 183

305 = 1.183 + 122

183 = 1.122 + 61

122 = 2.61 + 0,

diperoleh gcd(2013, 305) = gcd(305, 183) = gcd(183, 122) = gcd(122, 61) = 61.

Diperoleh 2013 = 61.33 dan 305 = 61.5. Akibatnya, peluang yang dimaksud

sama dengan peluang suatu bilangan kelipatan 5 habis dibagi 33, yaitu1

33. �

Contoh 4.3.2. Tentukan nilai dari

gcd(2014 + 2, 20142 + 2, 20143 + 2, . . .).

Penyelesaian. Misalkan d = gcd(2014+2, 20142+2, 20143+2, . . .). Diperhatikan

bahwa 20142+2 = 20142−4+6 = (2014−2)(2014+2)+6 = 2012(2014+2)+6.

Berdasarkan Algoritma Euclid diperoleh

gcd(2014 + 2, 20142 + 2) = gcd(2016, 6) = 6.

Akibatnya d|6. Di lain pihak, setiap bilangan pada barisan 2014 + 2, 20142 +

2, 20143+2, . . . habis dibagi 2. Lebih lanjut, karena 2014 = 2013+1 = 671.3+1,

maka untuk setiap bilangan bulat positif k berlaku 2014k = 3ak + 1 untuk suatu

bilangan bulat positif ak. Diperoleh 3|2014k+2 untuk setiap bilangan bulat posi-

tif k. Karena 2 dan 3 relatif prima, maka setiap bilangan pada barisan tersebut

habis dibagi oleh 6, sehingga diperoleh 6|d. Karena d|6 dan 6|d, maka d = 6. �

4.4 Identitas Bezout

Algoritma Euclid memberikan karakteristik penting terkait eksistensi penye-

lesaian persamaan linear dua variabel sebagai berikut.

Teorema 4.4.1 (Identitas Bezout). Untuk setiap bilangan bulat positif m dan n,

terdapat bilangan bulat x dan y dengan sifat mx+ ny = gcd(m,n).

38

Bukti. Berdasarkan Algoritma Euclid diperoleh bahwa

r1 = m− nq1, r2 = −mq2 + n(1 + q1q2), . . . .

Karena ri+1 = ri−1 + riqi+1, maka secara umum diperoleh ri = mαi + nβi untuk

i = 1, 2, . . . , k dengan

αi+1 = αi−1 + qi+1αi

βi+1 = βi−1 + qi+1βi

untuk i = 2, 3, . . . , k − 1. Akibatnya diperoleh gcd(m,n) = rk = αkm+ βkn. �

Identitas Bezout memberikan karakteristik terkait penyelesaian persamaan

berbentuk ax+ by = c.

Akibat 4.4.2. Diberikan bilangan bulat a, b, c. Persamaan ax+ by = c memiliki

penyelesaian bulat (x, y) jika dan hanya jika gcd(a, b) membagi c.

Identitas Bezout juga memberikan karakteristik lain terkait konsep keterba-

gian.

Teorema 4.4.3. Diberikan bilangan bulat positif a, b dan bilangan bulat c. Jika

a|bc dan gcd(a, b) = 1, maka a|c.

Bukti. Kasus c = 0 cukup jelas. Diasumsikan c = 0. Karena gcd(a, b) = 1,

maka berdasarkan Identitas Beout , ax + by = 1 untuk suatu bilangan bulat x

dan y. Akibatnya diperoleh acx = bcy = c. Karena a|acx dan a|bcy, maka a|c. �

Teorema 4.4.4. Diberikan bilangan bulat positif a, b yang relatif prima. Jika c

bilangan bulat dengan sifat a|c dan b|c, maka ab|c .

Bukti. Karena a|c, maka c = ax untuk suatu bilangan bulat x. Akibatnya b|ax.Karena gcd(a, b) = 1 dan b|ax, maka b|x. Diperoleh x = by untuk suatu bilangan

bulat y, sehingga didapat c = aby atau ab|c. �

Contoh 4.4.5. Tunjukkan bahwa untuk setiap bilangan prima p dan bilangan

bulat k dengan sifat 1 ≤ k < p berlaku p|(p

k

).

39

Penyelesaian. Diambil sebarang bilangan prima p dan bilangan bulat k dengan

sifat 1 ≤ k < p. Diperhatikan bahwa

k

(p

k

)= p

(p− 1

k − 1

).

Diperoleh bahwa p membagi k

(p

k

). Karena gcd(p, k) = 1, maka diperoleh bahwa

p membagi

(p

k

). �

4.5 Kelipatan Persekutuan Terkecil

Untuk setiap bilangan bulat positif k, didefinisikan Mk sebagai himpunan

semua kelipatan dari k. Berbeda dengan himpunan Dk yang didefinisikan se-

belumnya, Mk merupakan himpunan tak hingga.

Definisi 4.5.1. Diberikan bilangan bulat positif s dan t. Anggota terkecil dari

himpunan Ms ∩Mt disebut kelipatan persekutuan terkecil (least common

multiple) dari s dan t, dinotasikan dengan lcm(m,n).

Teorema 4.5.2. Diberikan bilangan bulat positif s dan t.

a. Jika lcm(s, t) = m, m = ss′ = tt′, maka gcd(s′, t′) = 1.

b. Jikam′ kelipatan persekutuan dari s dan t dan m′ = ss′ = tt′, gcd(s′, t′) =

1, maka m′ = lcm(s, t).

c. Jika m; kelipatan persekutuan dari s dan t, maka lcm(s, t)|m′.

d. Jika m|s dan n|s, maka lcm(m,n)|s.

e. Untuk setiap bilangan bulat n berlaku n.lcm(s, t) = lcm(ns, nt).

f. Jika s = pα11 . . . pαk

k dan t = pβ1

1 . . . pβk

k , αi, βi ≥ 0, i = 1, 2, . . . , k, maka

lcm(s, t) = pmax(α1,β1)1 . . . p

max(αk,βk)k .

Sifat berikut memberikan hubungan antara faktor persekutuan terbesar

dengan kelipatan persekutuan terkecil.

40

Teorema 4.5.3. Untuk sebarang bilangan bulat positif m dan n berlaku

mn = gcd(m,n).lcm(m,n).

Bukti. Misalkan m = pα11 . . . pαk

k dan n = pβ1

1 . . . pβk

k , αi, βi ≥ 0, i = 1, 2, . . . , k.

Berdasarkan Teorema 4.2.2 bagian e. dan Teorema 4.5.2 bagian f. diperoleh

gcd(m,n).lcm(m,n) = pmax(α1,β1)+min(α1,β1)1 . . . p

max(αk,βk)+min(αk,βk)k

= pα1+β1

1 . . . pαk+βk

k = mn.

Contoh 4.5.4. Diketahui a dan b bilangan bulat positif dengan a + b = 52 dan

lcm(a, b) = 168. Tentukan nilai dari ab.

Penyelesaian. Misalkan d = gcd(a, b). Diperoleh d|52 dan d|168, sehingga

d| gcd(52, 168). Karena 168 = 3.52+12, 52 = 4.12+4, 12 = 3.4, maka berdasarkan

Algoritma Euclid diperoleh gcd(168, 52) = 4, sehingga d|4. Diperhatikan bahwa

4|lcm(a, b), maka 4|a atau 4|b. Karena 4|a+b, maka 4|a dan |b, sehingga diperoleh4|d. Jadi, d = 4. Berdasarkan Teorema 4.5.3, diperoleh ab = 4.168 = 724. �

Lebih lanjut, untuk setiap bilangan bulat positif a1, a2, . . . , an, keli-

patan persekutuan terkecil dari a1, a2, . . . , an adalah bilangan bulat positif terkecil

yang merupakan kelipatan dari masing-masing a1, a2, . . . , an, dinotasikan dengan

lcm(a1, a2, . . . , an).

Soal Latihan

1. Buktikan bahwa untuk setiap bilangan bulat positif n, pecahann2 + n− 1

n2 + 2ntidak dapat disederhanakan.

2. Tentukan nilai dari2013∑k=1

gcd(k, 7).

41

3. Diberikan bilangan bulat positif a, b dan c. Tunjukkan bahwa

(a) gcd(ca, cb) = c gcd(a, b).

(b) gcd(a, bc) = gcd(a, gcd(a, b)c).

(c) gcd(a2, b2) = (gcd(a, b))2.

(d) jika gcd(a, b) = 1, maka gcd(a+ b, a2 − ab+ b2) = 1 atau 3.

4. Tentukan banyaknya bilangan bulat positif k dengan sifat lcm(66, 88, k) =

1212.

5. Tentukan semua pasangan bilangan bulat positif (a, b) yang memenuhi

gcd(a, b) + lcm(a, b) = a+ b+ 6.

6. Tentukan bilangan bulat positif m dan n yang memenuhi m2 + n2 = 85113

dan lcm(m,n) = 1764.

7. Tentukan banyaknya tripel bilangan bulat positif berurutan (a, b, c) dengan

sifat lcm(a, b) = 1000 dan lcm(b, c) = lcm(a, c) = 2000.

8. Tiga bilangan bulat positif a1 < a2 < a3 memenuhi gcd(a1, a2, a3) = 1

dan gcd(a1, a2), gcd(a2, a3), gcd(a3, a1) > 1. Tentukan nilai minimal yang

mungkin dari a1 + a2 + a3.

9. Diberikan bilangan bulat positif n. Tunjukkan bahwa jika n = pα11 pα2

2 . . . pαkk

faktorisasi prima dari n, maka terdapat sebanyak

(2α1 + 1)(2α2 + 1) . . . (2αk + 1)

pasangan bilangan bulat positif berbeda (a, b) dengan sifat lcm(a, b) = n.

10. Diberikan p1, p2, . . . , pk bilangan prima berbeda dan a1, a2, . . . , ak bilangan

bulat positif berbeda. Tentukan banyaknya cara memfaktorkan pa11 pa22 . . . pakk

menjadi perkalian dua bilangan bulat positif xy yang memenuhi x > y > 1

dan gcd(x, y) = 1.

11. Tunjukkan bahwa untuk setiap bilangan bulat positif a, b dan c berlaku

(lcm(a, b, c))2

lcm(a, b)lcm(a, c)lcm(b, c)=

(gcd(a, b, c))2

gcd(a, b) gcd(a, c) gcd(b, c).

42

12. Tunjukkan bahwa untuk setiap bilangan bulat positif a dan b berlaku jika

lcm(a, a+ 5) = lcm(b, b+ 5), maka a = b.

13. Diberikan bilangan bulat positif m dan n dengan m ganjil. Tunjukkan

bahwa

gcd(2m − 1, 2n + 1) = 1

14. Diberikan bilangan bulat positif n. Tentukan faktor persekutuan terbesar

dari (2n

1

),

(2n

3

), . . . ,

(2n

2n− 1

).

43

BAB V

KEKONGRUENEN

5.1 Pendahuluan

Pada bagian ini dibahas konsep kekongruenan dan kelas residu. Topik ini

menjadi bahan bahasan untuk Minggu ke-11. Beberapa teorema terkenal dalam

Teori Bilangan yang berkaitan dengan kekongruenan, seperti Teorema Euler dan

Teorema Kecil Fermat, diberikan pada bagian ini.

Setelah mempelajari topik bahasan pada bab ini yang meliputi modulo, kelas

residu:

1. Mahasiswa mampu menjelaskan konsep kekongruenan, kelas residu

2. Mahasiswa mampu membuktikan Teorema Euler dan Teorema Wilson

3. Mahasiswa mampu menerapkan konsep kongruensi beserta sifat-sifat untuk

memecahkan masalah yang berkaitan

5.2 Kekongruenan

Konsep kekogruenan pada bilangan bulat dikembangkan berdasarkan kon-

sep Algoritma Pembagian.

Definisi 5.2.1. Diberikan bilangan bulat a, b dan m dengan m = 0. Bilangan a

dan b dikatakan kongruen modulo m jika m membagi a−b, dinotasikan dengan

a ≡ b (mod m). Jika m tidak membagi a − b, maka bilangan a dan b dikatakan

tidak kongruen modulo m dan dinotasikan a ≡ b (mod m).

Relasi ”≡” pada definisi tersebut dinamakan relasi kongruensi. Beber-

apa karakteristik dasar terkait dengan kekongruenan diberikan sebagai berikut.

Teorema 5.2.2. Diberikan bilangan bulat a, b, c, d dan m.

a. a ≡ a (mod m).

b. Jika a ≡ b (mod m) dan b ≡ c (mod m), maka a ≡ c (mod m).

44

c. a ≡ b (mod m), maka b ≡ a (mod m).

d. Jika a ≡ b (mod m) dan c ≡ d (mod m), maka a + c ≡ b + d (mod m)

dan a− c ≡ b− d (mod m).

e. Jika a ≡ b (mod m), maka untuk setiap bilangan bulat k berlaku ka ≡ kb

(mod m).

f. Jika a ≡ b (mod m) dan c ≡ d (mod m), maka ac ≡ bd (mod m).

Secara umum, jika ai ≡ bi (mod m), i = 1, . . . , k, maka a1 . . . ak ≡b1 . . . bk (mod m). Lebih lanjut, jika a ≡ b (mod m), maka untuk setiap

bilangan bulat positif k berlaku ak ≡ bk (mod m).

g. a ≡ b (mod mi), i = 1, . . . , k jika dan hanya jika

a ≡ b (mod lcm(m1, . . . ,mk)).

Secara khusus, jika m1, . . . ,mk sepasang-sepasang relatif prima, maka

a ≡ b (mod mi), i = 1, . . . , k jika dan hanya jika a ≡ b (mod m1 . . .mk).

Contoh 5.2.3. Tentukan sisa pembagian 62013 oleh 37.

Penyelesaian. Diperhatikan bahwa 36 =≡ −1 (mod 7), maka diperoleh

62013 ≡ 6.62012 ≡ 6.(62)1006 ≡ 6.(−1)1006 ≡ 1 (mod 37).

Jadi, sisa pembagian 62013 oleh 37 adalah 6. �

Contoh 5.2.4. Tentukan dua digit terakhir dari 32013.

Penyelesaian. Diperhatikan bahwa

32013 = (35)40233 = (243)40227

≡ 4340227

≡ (1849)20127

≡ (49)20127

≡ (2401)10049.27

≡ (1)1001323

≡ 23 (mod 100).

45

Jadi, dua digit terakhir dari 32013 adalah 23. �

Contoh 5.2.5. Tunjukkan bahwa 7 habis membagi 32n+1 + 2n+2 untuk setiap

bilangan bulat positif n.

Penyelesaian. Diambil sebarang bilangan bulat positif n. Diperhatikan bahwa

32n+1 ≡ 3.9n ≡ 3.2n (mod 7) dan 2n+2 ≡ 4.2n (mod 7). Akibatnya

32n+1 + 2n+2 ≡ 7.2n ≡ 0 (mod 7).

Teorema 5.2.6. Diberikan bilangan bulat a, b dan n, n = 0 dengan sifat a =

nq1 + r1, b = nq2 + r2, 0 ≤ r1, r2 < |n|. a ≡ b (mod n) jika dan hanya jika

r1 = r2.

Bukti. Diperhatikan bahwa a−b = n(q1−q2)+(r1−r2), maka diperoleh n|(a−b)

jika dan hanya jika n|(r1 − r2). Karena |r1 − r2| < |n|, maka diperoleh n|(a− b)

jika dan hanya jika r1 = r2. �

Diperhatikan bahwa Teorema 3.2.4 dapat dinyatakan dalam konsep kekon-

gruenan sebagai berikut.

Akibat 5.2.7. Diberikan bilangan prima p. Jika x dan y bilangan bulat dengan

sifat xy ≡ 0 (mod p), maka x ≡ 0 (mod p) atau y ≡ 0 (mod p).

Hal ini merupakan salah satu contoh kesamaan yang terdapat dalam be-

berapa konsep teori bilangan: p|xy (notasi keterbagian), xy ≡ 0 (mod p) (notasi

kekongruenan) dan p = kxy (notasi persamaan Diophantine). Beberapa aplikasi

dari Teorema 4.4.3 dan Teorema 4.4.4 diberikan sebagai berikut.

Akibat 5.2.8. Diberikan bilangan bulat positif m dan bilangan bulat a, b dan c

dengan c = 0. Jika ac ≡ bc (mod m), maka

a ≡ b (modm

gcd(c,m)).

46

Akibat 5.2.9. Diberikan bilangan bulat positif m dan a bilangan bulat yang relatif

prima dengan m. Jika a1 dan a2 bilangan bulat dengan sifat a1 ≡ a2 (mod m),

maka a1a ≡ a2a (mod m).

Contoh 5.2.10. Tentukan semua bilangan prima p dan q dengan sifat p + q =

(p− q)3.

Penyelesaian. Misalkan bilangan prima p dan q memenuhi p + q = (p − q)3.

Diperhatikan bahwa (p−q)3 = p+q = 0, diperoleh p = q yang berarti p dan q re-

latif prima. Karena p−q ≡ 2p (mod p+q), maka diperoleh 0 ≡ 8p3 (mod p+q).

Karena p dan q relatif prima, maka p dan p+ q relatif prima, sehingga diperoleh

0 ≡ 8 (mod p + q). Artinya p + q|8. Dapat dicek bahwa bilangan prima (p, q)

yang memenuhi hanya (3, 5) atau (5, 3). �

Berikut diberikan sifat yang bermanfaat dalam menyederhanakan ben-

tuk pangkat pada relasi kongruensi.

Teorema 5.2.11. Diberikan bilangan bulat m, a dan b dengan a dan b relatif

prima terhadap m. Jika x dan y bilangan bulat positif dengan sifat

ax ≡ bx (mod m) dan ay ≡ by (mod m),

maka

agcd(x,y) ≡ bgcd(x,y) (mod m).

Bukti. Berdasarkan Identitas Bezout, terdapat bilangan bulat tak negatif u dan

v dengan sifat gcd(x, y) = ux− vy. Diperoleh

aux ≡ bux (mod m) dan avy ≡ bvy (mod m),

sehingga berlaku auxbvy ≡ avybux mod m. Karena gcd(a,m) = gcd(m,n) = 1,

maka diperoleh

agcd(x,y) ≡ aux−vy ≡ bux−vy ≡ bgcd(x,y) (mod m).

47

5.3 Kelas Residu

Berdasarkan Teorema 5.2.2 bagian a. b. dan c., diperoleh bahwa untuk

sebarang bilangan bulat positif m, setiap bilangan bulat dapat diklasifikasikan

secara tunggal ke dalam suatu kelas berdasarkan sisanya ketika dibagi oleh m.

Jelas bahwa terdapat sebanyak m kelas.

Definisi 5.3.1. Diberikan bilangan bulat positif n. Himpunan bilangan bulat S

disebut himpunan kelas residu lengkap modulo n jika untuk setiap i dengan

0 ≤ i ≤ n− 1, terdapat s ∈ S dengan sifat i ≡ s mod n.

Diperhatikan bahwa {a, a+1, a+2, . . . , a+m−1} merupakan himpunan

kelas residu lengkap modulo m untuk sebarang bilangan bulat a.

Contoh 5.3.2. Diberikan bilangan bulat positif n. Pernyataan-pernyataan dibawah

ini benar.

a. n2 ≡ 0 atau 1 (mod 3);

b. n2 ≡ 0 atau ±1 (mod 5);

c. n2 ≡ 0 atau 1 atau 4 (mod 8);

d. n3 ≡ 0 atau ±1 (mod 9);

e. n3 ≡ 2 atau 3 atau 5 (mod 7);

f. n4 ≡ 0 atau 1 (mod 16).

Bukti diserahkan sebagai latihan.

Contoh 5.3.3. Tunjukkan bahwa tidak ada bilangan bulat x dan y yang memenuhi

x2 − 5y2 = 2013.

Penyelesaian. Diandaikan bilangan bulat x dan y memenuhi x2 − 5y2 = 2013.

Diperhatikan bahwa x2 − 5y2 ≡ 0 atau ±1 (mod 5). Di sisi lain, 2013 ≡ 3

(mod 5), suatu kontradiksi. �

48

Contoh 5.3.4. Diberikan m bilangan genap positif. Diasumsikan bahwa

{a1, a2, . . . , am} dan {b1, b2, . . . , bm}

dua himpunan kelas residu lengkap modulo m. Tunjukkan bahwa

{a1 + b1, a2 + b2, . . . , am + bm}

bukan himpunan kelas residu lengkap modulo m.

Penyelesaian. Diandaikan {a1+ b1, a2+ b2, . . . , am+ bm} himpunan kelas residu

lengkap modulo m. Diperoleh

1 + 2 + . . .+ n ≡ (a1 + b1) + (a2 + b2) + . . .+ (am + bm)

≡ (a1 + a2 + . . .+ am) + (b1 + b2 + . . .+ bm)

≡ 2(1 + 2 + . . .+m) (mod m),

sehingga 1 + 2 + . . . + m ≡ 0 (mod m) atau m|m(m+ 1)

2. Kontradiksi den-

gan fakta bahwa untuk bilangan genap m, m |m(m+ 1)

2. Jadi, {a1 + b1, a2 +

b2, . . . , am + bm} bukan himpunan kelas residu lengkap modulo m. �

Teorema 5.3.5. Diberikan bilangan bulat positif m dan a, b bilangan bulat dengan

gcd(a,m) = 1. Jika S himpunan kelas residu lengkap modulo m, maka

T = aS + b = {as+ b : s ∈ S}

merupakan himpunan kelas residu lengkap modulo m.

Bukti. Diketahui S himpunan kelas residu lengkap modulo m. Diperoleh bahwa

banyak anggota dari S ada sebanyak m, misalkan S = {s1, s2, . . . , sm} den-

gan si ≡ sj, i = j dan untuk setiap i = 0, 1, 2, . . . ,m − 1 terdapat j dengan

sifat sj ≡ i mod m. Diperhatikan bahwa T = {as + b : s ∈ S}, maka diper-

oleh banyak anggota dari T ada sebanyak m, sehingga cukup ditunjukkan setiap

anggotanya tidak kongruen satu sama lain dalam modulo m. Diandaikan terda-

pat asi + b ≡ asj + b (mod m) untuk suatu 1 ≤ i < j ≤ m. Diperoleh asi ≡ asj

(mod m). Karena gcd(a,m) = 1, maka diperoleh si ≡ sj (mod m). Kontradiksi

49

dengan fakta si ≡ sj, i = j. Jadi, T merupakan himpunan kelas residu lengkap

modulo m. �

Selanjutnya, diberikan hubungan antara kelas residu dengan persamaan

kongruensi linear.

Teorema 5.3.6. Diberikan bilangan bulat positif m. Jika a, b bilangan bulat den-

gan gcd(a,m) = 1, maka terdapat bilangan bulat x dengan sifat ax ≡ b (mod m)

dan semua bilangan x yang memenuhi kondisi tersebut berada pada tepat satu

kelas residu modulo m.

Bukti. Misalkan {c1, c2 . . . , cm} himpunan kelas residu lengkap modulom. Berdasarkan

Teorema 5.3.5,

{ac1 − b, ac2 − b, . . . , acm − b}

merupakan himpunan kelas residu lengkap. Akibatnya, terdapat ci dengan sifat

aci − b ≡ 0 (mod m), dengan kata lain ci merupakan solusi persamaan kongru-

ensi ax ≡ b (mod m). Lebih lanjut, jika x dan x′ solusi persamaan kongruensi

ax ≡ b (mod m), maka berlaku ax ≡ ax′ (mod m). Karena gcd(a,m) = 1, maka

diperoleh x ≡ x′ (mod m). �

Khusus untuk b = 1, pada Teorema 5.3.6 diperoleh bahwa jika gcd(a,m) =

1, maka terdapat x dengan sifat ax ≡ 1 (mod m). Bilangan x tersebut disebut

invers dari a modulo m, dinotasikan dengan a−1 atau 1a(mod m). Karena se-

mua bilangan x yang memenuhi kondisi ax ≡ 1 (mod m) berada pada tepat satu

kelas residu modulo m, maka invers dari a modulo m terdefinisi dengan baik.

Teorema 5.3.7 (Teorema Wilson). Untuk setiap bilangan prima p berlaku (p−1)! = −1 (mod p).

Bukti. Untuk kasus p = 2 dan p = 3 cukup jelas. Diambil sebarang bilangan

prima p ≥ 5. Misalkan S = {2, 3, . . . , p − 2}. Karena p prima, maka untuk

sebarang s ∈ S memiliki invers tunggal s′ ∈ {1, 2, . . . , p − 1}. Lebih lanjut,

s′ = 1 dan s′ = p − 1, akibatnya s′ ∈ S. Diperhatikan bahwa s′ = s sebab jika

s′ = s, maka s2 ≡ 1 (mod p), sehingga diperoleh p|s−1 atau p|s+1. Hal ini tidak

50

mungkin sebab s+1 < p. Akibatnya diperoleh bahwa anggota-anggota dari S da-

pat dipartisi menjadi p−32

pasangan berbeda (s, s′) dengan sifat ss′ ≡ 1 (mod p).

Dengan mengalikan p−32

persamaan kongruensi tersebut diperoleh (p − 2)! ≡ 1

(mod p), sehingga didapat (p− 1)! ≡ −1 (mod p). �

Diperhatikan bahwa konvers dari Teorema Wilson benar, yaitu jika

(n − 1)! ≡ −1 (mod n) untuk suatu bilangan bulat positif n ≥ 2, maka n

prima, sebab jika n = n1n2 untuk suatu bilangan bulat positif n1, n2 ≥ 2, maka

n1|1.2. . . . n1 . . . (n − 1) + 1, suatu kontradiksi. Hal ini memberikan cara lain

mengetahui suatu bilangan merupakan bilangan prima atau tidak. Namun, un-

tuk n yang cukup besar hal ini sulit dilakukan.

Contoh 5.3.8. Diberikan p bilangan prima dengan p ≡ 1 (mod 4). Tunjukkan

bahwa [(p− 1

2

)!

]2≡ −1 (mod p).

Penyelesaian. Berdasarkan Teorema Wilson diperoleh

−1 ≡ (p−1)! ≡∏

1≤i≤(p−1)/2

i(p−i) ≡∏

1≤i≤(p−1)/2

−i2 ≡ (−1)(p−1)/2

[(p− 1

2

)!

]2(mod p).

Karena p ≡ 1 (mod 4), maka (−1)(p−1)/2 = 1. Jadi,[(p− 1

2

)!

]2≡ −1 (mod p).

5.4 Teorema Kecil Fermat dan Teorema Euler

Untuk setiap bilangan bulat positif m, banyaknya bilangan bulat positif

n yang kurang dari m dan relatif prima dengan m dinotasikan dengan φ(n).

Fungsi φ disebut fungsi Euler. Jelas bahwa φ(1) = 1 dan untuk setiap bilangan

prima p, φ(p) = p − 1. Lebih lanjut, jika n bilangan bulat positif dengan sifat

φ(n) = n − 1, maka n prima. Beberapa karakteristik lain dari fungsi Euler

diberikan sebagai berikut.

51

Teorema 5.4.1. Untuk setiap bilangan prima p dan bilangan bulat positif a

berlaku φ(pa) = pa − pa−1.

Teorema 5.4.2. Jika a dan b bilangan bulat positif yang relatif prima, maka

φ(ab) = φ(a)φ(b).

Bukti. Disusun bilangan 1, 2, . . . , ab sebagai berikut:

1 2 . . . aa+ 1 a+ 2 . . . 2a...

......

...a(b− 1) + 1 a(b− 1) + 2 . . . ab.

Jelas bahwa di antara bilangan-bilangan 1, 2, . . . , ab terdapat φ(ab) bilangan yang

relatif prima dengan ab. Di lain pihak, terdapat φ(a) kolom yang mengandung

bilangan-bilangan yang relatif prima dengan a. Karena setiap kolom merupakan

himpunan kelas residu lengkap modulo b, maka terdapat tepat φ(b) bilangan pada

masing-masing kolom yang relatif prima dengan b. Akibatnya, banyaknya bilan-

gan yang relatif prima dengan ab pada susunan tersebut adalah φ(a)φ(b). Jadi,

φ(ab) = φ(a)φ(b). �

Berdasarkan Teorema 5.4.1 dan Teorema 5.4.2 diperoleh karakteristik

nilai fungsi Euler dari setiap bilangan bulat positif.

Teorema 5.4.3. Diberikan bilangan bulat positif n. Jika n = pα11 pα2

2 . . . pαkk fak-

torisasi prima dari n, maka

φ(n) = n

(1− 1

p1

)(1− 1

p2

). . .

(1− 1

pk

).

Contoh 5.4.4. Tunjukkan bahwa ada tak hingga banyaknya bilangan bulat positif

n dengan sifat 10|φ(n).

Penyelesaian. Diambil n = 11k, k = 1, 2, . . .. Diperoleh φ(11k) = 11k−11k−1 =

10.11k−1. �

Definisi 5.4.5. Diberikan bilangan bulat positif m. Himpunan bilangan bulat

S disebut himpunan kelas residu tereduksi lengkap modulo m jika untuk

setiap 0 ≤ i ≤ n − 1 dengan gcd(i,m) = 1, terdapat s ∈ S dengan sifat i ≡ s

(mod m).

52

Jelas bahwa suatu himpunan kelas residu tereduksi lengkap modulo m

memiliki sebanyak φ(m) anggota.

Teorema 5.4.6. Diberikan bilangan bulat positif m dan a dengan gcd(a,m) = 1.

Jika S himpunan kelas residu tereduksi lengkap modulo m, maka himpunan

T = aS = {as|s ∈ S},

merupakan himpunan kelas residu tereduksi lengkap modulo m.

Diberikan bilangan bulat positif m dan S = {a1, a2, . . . , aφ(m)} him-

punan kelas residu tereduksi lengkap modulo m. Berdasarkan eksistensi dan

ketunggalan invers dalam modulo m, dapat ditunjukkan bahwa himpunan invers

anggota-anggota dari S, dinotasikan dengan

{a−11 , a−1

2 , . . . , a−1φ(m)} atau

{1

a1,1

a2, . . . ,

1

aφ(m)

},

merupakan himpunan kelas residu tereduksi lengkap modulo m.

Teorema 5.4.7 (Teorema Euler). Jika a dan m bilangan bulat positif dengan

gcd(a,m) = 1, maka aφ(m) ≡ 1 (mod m).

Bukti. Misalkan S = {a1, a2, . . . , aφ(m)} himpunan semua bilangan bulat positif

yang kurang dari m dan relatif prima dengan m. Karena gcd(a,m) = 1, maka

berdasarkan Teorema 5.4.6 berlaku

{aa1, aa2, . . . , aaφ(m)}

merupakan himpunan kelas residu tereduksi lengkap modulom. Akibatnya diper-

oleh

(aa1)(aa2) . . . (aaφ(m)) ≡ a1a2 . . . aφ(m) (mod m).

Karena gcd(ak,m) = 1, k = 1, 2, . . . , φ(m), maka diperoleh

aφ(m) ≡ 1 (mod m).

Dengan mengambil m bilangan prima, maka Teorema Euler menjadi

Teorema Kecil Fermat.

53

Teorema 5.4.8 (Teorema Kecil Fermat). Jika a bilangan bulat positif dan p

bilangan prima, maka ap ≡ a (mod p).

Contoh 5.4.9. Diberikan bilangan prima p ≥ 7. Tunjukkan bahwa

11 . . . 1︸ ︷︷ ︸(p−1) kali

habis dibagi oleh p.

Penyelesaian. Diperhatikan bahwa

11 . . . 1︸ ︷︷ ︸(p−1) kali

=10p−1 − 1

9.

Karena gcd(10, p) = 1, maka berdasarkan Teorema Kecil Fermat diperoleh p|10p−1−1, sehingga

11 . . . 1︸ ︷︷ ︸(p−1) kali

habis dibagi oleh p. �

Contoh 5.4.10. Diberikan bilangan prima p > 5. Tunjukkan bahwa p8 ≡ 1

(mod 240).

Penyelesaian. Diperhatikan bahwa 240 = 24.3.5. Berdasarkan Teorema Ke-

cil Fermat, p2 ≡ 1 (mod 3) dan p4 ≡ 1 (mod 5). Karena gcd(p, 24) = 1 dan

φ(24) = 8, maka berdasarkan Teorema Euler diperoleh p8 ≡ 1 (mod 16). Jadi,

p8 ≡ 1 (mod m) untuk m = 3, 5 dan 16. Akibatnya p8 ≡ 1 (mod 240). �

Berdasarkan Teorema Euler diperoleh jika a dan m bilangan bulat posi-

tif yang relatif prima, maka terdapat bilangan bulat positif x dengan sifat ax ≡ 1

(mod m).

Definisi 5.4.11. Diberikan bilangan bulat positif m. Bilangan bulat positif a

dikatakan memiliki order d modulo m, dinotasikan ordm(a) = d, jika d adalah

bilangan bulat positif terkecil dengan sifat ad ≡ 1 (mod m).

54

Berdasarkan Teorema Euler, ordm(a) = d ≤ φ(m). Jika bilangan bulat

positif x memenuhi ax ≡ 1 (mod m), maka berdasarkan Teorema 5.2.11,

agcd(x,d) ≡ 1 (mod m).

Karena gcd(x, d) ≤ d dan d bilangan bulat positif terkecil dengan sifat ad ≡1 (mod m), maka diperoleh gcd(x, d) = d. Artinya, d membagi x, sehingga

diperoleh Teorema sebagai berikut.

Teorema 5.4.12. Bilangan bulat positif x memenuhi ax ≡ 1 (mod m) jika dan

hanya jika x kelipatan dari order a modulo m.

Contoh 5.4.13. Tentukan order dari 8 modulo 11.

Penyelesaian. Berdasarkan Teorema Kecil Fermat, diperoleh 810 ≡ 1 (mod 11).

Akibatnya, ord11(8)|10. Diperhatikan bahwa 82 ≡ −2 (mod 11)dan 85 ≡ −1

(mod 11). Jadi, ord11(8) = 10. �

Soal Latihan

1. Tunjukkan bahwa 7|22225555 + 55552222.

2. Berapakah sisa pembagian 434343

oleh 100?

3. Tentukan digit ratusan dari 20132013.

4. Tentukan semua bilangan bulat n1, n2, . . . , n12 yang memenuhi n41 + n4

2 +

. . .+ n412 = 2013.

5. Tentukan sisa pembagian 683 + 883 oleh 49.

6. Tunjukkan bahwa tidak ada bilangan bulat positif x dan y dengan sifat

x3 = 2y + 15.

7. Tentukan order dari 5 modulo 12.

8. Tunjukkan bahwa untuk setiap bilangan bulat positif n berlaku n9 ≡ n3

(mod 504).

55

9. Diberikan p dan q bilangan prima berbeda. Tunjukkan bahwa untuk setiap

bilangan bulat a berlaku

pq|(apq − ap − aq + a).

10. Tunjukkan bahwa untuk setiap bilangan genap positif n berlaku n2|2n! − 1.

11. Tunjukkan bahwa untuk setiap bilangan prima p > 5 berlaku p4 ≡ 1

(mod 240).

12. (a) Tentukan jumlah semua bilangan bulat positif yang kurang dari 2013

dan relatif prima dengan 2013.

(b) Tentukan jumlah semua bilangan bulat positif yang kurang dari 4026

dan relatif prima dengan 2013.

13. Tentukan banyaknya bilangan bulat positif m < 2013 dengan sifat

{2013, 4026, 6039, . . . , 2013m}

merupakan himpunan kelas residu lengkap modulo m.

14. Diberikan p bilangan prima. Tunjukkan bahwa p membagi abp − bap untuk

setiap bilangan bulat positif a dan b.

15. Tunjukkan bahwa untuk setiap bilangan bulat positif n berlaku∑d|n

φ(d) = n.

16. Tunjukkan bahwa untuk setiap bilangan prima ganjil p berlaku

12.32. . . . (p− 2)2 ≡ 22.42 . . . (p− 1)2 ≡ (−1)(p−1)/2 (mod p).

56

BAB VI

PERSAMAAN LINEAR DIOPHANTINE

6.1 Pendahuluan

Pada bagian ini dibahas konsep persamaan linear Diophantine dan eksis-

tensi solusi bulat dari persamaan tersebut. Topik ini merupakan pokok bahasan

pada Minggu ke-11 dan 12. Lebih lanjut, diberikan karakteristik solusi bulat

non-negatif persamaan linear Diophantine dua variabel.

Setelah mempelajari bab ini diharapkan mahasiswa memiliki learning out-

comes berupa:

1. Mahasiswa mampu menjelaskan pengertian Diophantine Linear

2. Mahasiswa mampu menyelesaikan soal-soal yang berkaitan dengan Dio-

phantine Linear

6.2 Persamaan Linear Diophantine

Definisi 6.2.1. Diberikan bilangan bulat positif n dan a1, a2, . . . , an, b bilangan

bulat dengan ai = 0, i = 1, 2, . . . , n. Persamaan

a1x1 + a2x2 + · · ·+ anxn = b, (6.1)

disebut persamaan linear Diophantine.

Berikut diberikan syarat cukup dan perlu agar persamaan (6.1) memiliki

solusi bulat.

Teorema 6.2.2. Persamaan (6.1) memiliki solusi bulat jika dan hanya jika

gcd(a1, a2, . . . , an)|b.

Lebih lanjut, jika persamaan (6.1) memiliki solusi bulat, maka semua solusi bulat

dari persamaan (6.1) dapat dinyatakan ke dalam n− 1 parameter.

57

Bukti. Misalkan d = gcd(a1, a2, . . . , an).

Jika b tidak habis dibagi oleh d,maka persamaan (6.1) tidak memiliki solusi bulat

sebab untuk setiap bilangan bulat x1, x2, . . . , xn, ruas kiri dari persamaan (6.1)

habis dibagi oleh d sedangkan ruas kanannya tidak.

Jika d|b, maka diperoleh persamaan (6.1) ekuivalen dengan

a′1x1 + a′2x2 + · · ·+ a′nxn = b′, (6.2)

dengan a′i = ai/d, ı = 1, 2, . . . , n dan b′ = b/d. Jelas bahwa gcd(a′1, a′2, . . . , a

′n) =

1. Akan digunakan induksi matematika untuk membuktikan persamaan (6.2)

memiliki solusi bulat.

Kasus n = 1. Persamaan (6.2) menjadi x1 = b atau −x1 = b. Diperoleh

x1 = b atau x1 = −b merupakan solusi, dan solusi ini tidak berisi parameter.

Diasumsikan persamaan (6.2) dengan n−1 variabel memiliki solusi bulat(n ≥2). Akan dibuktikan bahwa persamaan (6.1) dengan n variabel memiliki solusi

bulat. Misalkan dn−1 = gcd(a′1, a′2, . . . , a

′n−1). Diperoleh bahwa setiap solusi bulat

dari persamaan (6.2) memenuhi

a′1x1 + a′2x2 + · · ·+ a′nxn ≡ b′ (mod dn−1),

yang ekuivalen dengan

a′nxn ≡ b′ (mod dn−1). (6.3)

Dengan mengalikan kedua ruas pada persamaan (6.2) dengan a′nφ(dn−1)−1 dan

menggunakan fakta bahwa a′nφ(dn−1) ≡ 1 (mod dn−1), diperoleh

xn ≡ c (mod dn−1),

dengan c = a′nφ(dn−1)−1b′. Artinya, xn = c+ dn−1tn−1 untuk suatu bilangan bulat

tn−1. Dengan mensubstitusi xn ke persamaan (6.2), diperoleh

a′1x1 + · · ·+ a′n−1xn−1 = b− a′nc− a′ndn−1tn−1.

Diperhatikan bahwa dn−1|(b′ − a′nc − a′n−1dn−1tn−1) atau yang ekuivalen dengan

a′nc ≡ b′ (mod dn−1), sebab c = a′nφ(dn−1)−1b′. Akibatnya, dengan membagi kedua

ruas persamaan terakhir dengan dn−1 diperoleh

a”1x1 + a”2x2 + · · ·+ a”n−1xn−1 = b” (6.4)

58

dengan a”i = a′i/dn−1 untuk i = 1, 2, . . . , n dan b” = (b′ − a′nc)/dn−1 + a′ntn−1.

Karena gcd(a”1, a”2, . . . , a”n−1) = 1 maka sesuai asumsi induksi, persamaan (6.4)

memiliki solusi bulat dan solusinya dapat ditulis ke dalam n− 2 parameter. Jika

pada solusi tersebut ditambahkan xn = c + dn−1tn−1 maka diperoleh persamaan

(6.2) memiliki solusi bulat dan solusinya dapat ditulis ke dalam n− 1 parameter.

Contoh 6.2.3. Tentukan bilangan bulat x dan y yang memenuhi persamaan

a. 96x+ 54y = 20.

b. 96x+ 54y = 12.

Penyelesaian. Berdasarkan Algoritma Euclid diperoleh

96 = 1.54 + 42

54 = 1.42 + 12

42 = 3.12 + 6

12 = 2.6 + 0.

Diperoleh gcd(96, 54) = 6.

a. Karena 6 |20, maka berdasarkan Teorema 6.2.2 persamaan 96x + 54y = 20

tidak memiliki solusi bulat.

b. Karena 6|12, maka berdasarkan Teorema 6.2.2 persamaan 96x + 54y = 20

memiliki solusi bulat. Akan dicari salah satu solusinya menggunakan Algo-

ritma Euclid. Diperhatikan bahwa

6 = 42− 3.12

12 = 54− 42

42 = 96− 54.

Diperoleh

6 = 42− 3(54− 42)

= 4.42− 3.54

= 4(96− 54)− 3.54

= 4.96− 7.54,

59

sehingga didapat 12 = 8.96 + (−14)54. Jadi, x = 8 dan y = −14 memenuhi

persamaan 96x+ 54y = 12.

Teorema 6.2.2 memberikan suatu karakteristik dari setiap solusi bulat

persamaan berbentuk ax+ by = c.

Teorema 6.2.4. Diberikan a, b dan c bilangan bulat dengan gcd(a, b)|c. Jika

(x0, y0) merupakan solusi bulat dari persamaan

ax+ by = c,

maka setiap solusi bulat dari persamaan tersebut dinyatakan dalam bentuk

x = x0 +b

gcd(a, b)t, y = y0 −

a

gcd(a, b)t

untuk suatu bilangan bulat t.

Penyelesaian. Misalkan a = gcd(a, b)a′ dan b = gcd(a, b)b′. Diperoleh gcd(a′, b′) =

1. Diketahui (x0, y0) merupakan solusi bulat dari persamaan ax+ by = c. Diper-

oleh ax0 + by0 = c. Diambil sebarang (x, y) solusi bulat persamaan ax+ by = c.

Diperoleh

ax+ by = c = ax0 + by0

⇐⇒ a(x− x0) = b(y0 − y).

Dengan membagi kedua ruas persamaan tersebut dengan gcd(a, b) diperoleh

a′(x− x0) = b′(y0 − y).

Diperhatikan bahwa a′|a′(x − x0), maka a′|b′(y0 − y). Karena gcd(a′, b′) = 1,

maka a′|y0− y. Artinya, terdapat bilangan bulat s dengan sifat y0− y = a′s atau

y = y0 − a′s. Dengan cara yang sama, terdapat bilangan bulat t dengan sifat

x = x0 + b′t. Dengan mensubstitusi x dan y ke persamaan terakhir diperoleh

s = t. Jadi,

x = x0 +b

gcd(a, b)t, y = y0 −

a

gcd(a, b)t.

60

Contoh 6.2.5. Tentukan semua solusi bulat persamaan 96x+ 54y = 12.

Penyelesaian. Diperhatikan bahwa x = 8 dan y = −14 merupakan salah satu

solusi persaman 96x + 54y = 12. Akibatnya diperoleh setiap solusi persamaan

96x+ 54y = 12 berbentuk

x = 8 + 9t, y = −14− 16t

untuk suatu bilangan bulat t. �

Contoh 6.2.6. Tentukan semua triple(x, y, z) yang memenuhi persamaan 3x +

4y + 5z = 2013.

Penyelesaian. Dari persamaan tersebut diperoleh 3x+ 4y ≡ 3 mod 5 ,artinya

3x+ 4y = 3 + 5s

untuk suatu bilangan bulat s .Salah satu solusi dari persamaan tersebut adalah

: x = 1 + 3s ,y = −s. Berdasarkan Teorema 6.2.4, diperoleh x = 1 + 3s + 4t

,y = −s − 3t untuk suatu bilangan bulat t. Dengan mensubstitusi x dan y ke

persamaan awal, didapat z = 402− s.

Jadi, semua solusinya adalah

(x, y, z) = (1 + 3s+ 4t,−s− 3t, 402− s)

untuk setiap pasangan bilangan bulat s, t. �

Contoh 6.2.7. Diberikan bilangan bulat positif n. Diasumsikan terdapat 666

triple berurutan bilangan bulat positif (x, y, z) yang memenuhi persamaan

x+ 2010y + 2010z = n.

Tentukan nilai maksimum dan minimum dari n.

Penyelesaian. Jawabannya maksimum 76379 dan minimum 74370.

Misalkan n = 2010a+b dengan a dan b bilangan bulat dan 0 ≤ b < 2010. Karena

x ≡ n ≡ b (mod 2010), maka nilai yang mungkin untuk x adalah b, 2010+ b, . . . ,

61

2010(a − 1) + b. Untuk x = b + 2010i, 0 ≤ i ≤ a − 1, diperoleh 2010(y + z) =

2010(a− i) atau y+ z = a− i. Persamaan y+ z = a− i memiliki a− i− 1 solusi

bilangan bulat positif (y, z), yaitu (1, a − i − 1), . . . , (a − i − 1, 1). Akibatnya

terdapata−1∑i=0

(a− i− 1) =a−1∑i=0

i =a(a− 1)

2

triple berurutan yang memenuhi kondisi yang ditentukan. Dari persamaan a(a−1)2

=

666, diperoleh a = 37. Jadi, nilai maksimum dari n sama dengan 37·2010+2009 =

76379 (diperoleh dengan mengambil b = 2009 ) dan nilai minimum dari n sama

dengan 37 · 2010 = 74370 (diperoleh dengan mengambil b = 0). �

6.3 Teorema Frobenius

Diperhatikan bahwa jika a dan b bilangan bulat positif dengan gcd(a, b) = 1,

maka untuk setiap bilangan bulat n persamaan ax+by = n selalu memiliki solusi

bulat. Berikut diberikan karakteristik yang menjamin eksistensi solusi bulat non-

negatif dari persamaan ax+ by = n.

Teorema 6.3.1 (Teorema Frobenius). Diberikan bilangan bulat positif a dan b.

Jika gcd(a, b) = 1, maka banyaknya bilangan bulat positif m yang tidak dapat

dinyatakan ke dalam bentuk ar + bs = m untuk suatu bilangan bulat non-negatif

r dan s sama dengan(a− 1)(b− 1)

2.

Bukti. Misalkan bilangan bulat positif n dikatakan ”baik” jika terdapat bilangan

bulat non-negatif r dan s dengan sifat ar+bs = n. Diperhatikan susunan berikut:

0 1 2 . . . k . . . a− 1a a+ 1 a+ 2 . . . a+ k . . . 2a− 12a 2a+ 1 2a+ 2 . . . 2a+ k . . . 3a− 1. . . . . . . . . . . . . . . . . . . . .

Setiap kolom membentuk barisan artimatik dengan beda a. Diperhatikan bahwa

jika n baik, maka n+ ka baik untuk setiap bilangan bulat positif k. Jelas bahwa

setiap kelipatan dari b baik. Diperhatikan bahwa tidak ada dua kelipatan dari

b, vb dan wb dengan 0 ≤ v, w ≤ a − 1, berada pada kolom yang sama sebab

62

jika tidak, maka vb ≡ wb (mod a). Akibatnya b(v − w) ≡ 0 (mod a). Karena

gcd(a, b) = 1, maka v − w ≡ 0 (mod a). Karena 0 ≤ v, w ≤ a− 1, maka v = w.

Selanjutnya, akan ditunjukkan setiap bilangan yang berada tepat di atas salah

satu kelipatan vb, 0 ≤ v ≤ a − 1, tidak baik. Sebarang bilangan yang berada

tepat di atas vb berbentuk vb − ka untuk suatu bilangan bulat positif k. Jika

bv − ka baik, maka ax + by = bv − ka untuk suatu bilangan bulat non-negatif

x dan y. Diperoleh by ≤ ax + by = bv − ka ≤ bv, yang berarti 0 ≤ y < v < a.

Akibatnya y ≡ v (mod a). Di sisi lain, dua bilangan yang terletak pada kolom

yang sama kongruen modulo a. Diperoleh vb ≡ vb − ka ≡ ax + by (mod a),

yang berarti bv ≡ by (mod a). Karena gcd(a, b) = 1, maka v ≡ y (mod a), suatu

kontradiksi.

Diperoleh banyaknya bilangan yang tidak baik sama dengan banyaknya bilangan

yang berada tepat di atas bilangan berbentuk vb, 0 ≤ v ≤ a − 1. Diperhatikan

bahwa pada kolom ke j, terdapat (vb − j)/a bilangan yang berada di atas vb.

Akibatnya diperoleh banyaknya bilangan yang tidak baik adalah

a−1∑v=0

a−1∑j=0

vb− j

a=

(a− 1)(b− 1)

2.

Diperhatikan bahwa bilangan bulat positif terbesar yang tidak baik be-

rada tepat di atas bilangan (a − 1)b, sehingga diperoleh bilangan terbesar yang

tidak baik adalah (a− 1)b− a.

Teorema 6.3.2. Diberikan bilangan bulat positif a dan b yang saling prima.

Persamaan

ax+ by = n

tidak memiliki solusi bulat non-negatif x dan y untuk n = ab − a − b. Jika

n > ab− a− b, maka persamaan tersebut memiliki solusi bulat non-negatif x dan

y.

Contoh 6.3.3. Diketahui n bilangan bulat positif dengan gcd(n, 1991) = 1. Tun-

jukkan abhwan

1991dapat ditulis sebagai jumlahan dua bilangan rasional positif

dengan penyebut kurang dari 1991 jika dan hanya jika terdapat bilangan bulat

positif m, a, b dengan m ≤ 10 dan mn = 11a+ 181b.

63

Penyelesaian. (⇐) Diketahui m, a, b bilangan bulat positif dengan m ≤ 10 dan

mn = 11a+ 181b. Diperoleh

n

1991=

a

181m+

b

11m,

dengan 181m, 11m < 1991.

(⇒)Diketahuin

1991=

a

r+

b

runtuk suatu bilangan bulat positif a, b dengan gcd(a, r) =

gcd(b, s) = 1 dan r, s < 1991. Tanpa mengurangi keumuman misalkan r = 181r1

dan s = 11s1 (1991 = 11.181). Diperoleh nr1s1 = 11as1 + 181br1. Akibatnya

r1|11as1 dan s1|181br1. Karena r1 < 11 dan gcd(r1, a) = 1, maka r1|s1. Karena

s1 < 181 dan gcd(s1, b) = 1, maka s1|r1. Diperoleh s1 = r1. Dipilih m = r1. Jelas

m merupakan bilangan bulat positif yang kurang dari 11 dan mn = 11a+181b. �

Soal Latihan

1. Tentukan semua solusi bulat persamaan-persamaan linear Diophantine berikut.

(a) 20x+ 13y = 2013.

(b) 3456x+ 246y = 44.

(c) 3u+ 4v + 5w + 6x = 18.

2. Tentukan semua solusi bulat persamaan

(6u+ 9v)(7x+ 8y) = 3.

3. Tentukan semua bilangan bulat x dan y dengan sifat

3x+ 4y|2013.

4. Tunjukkan bahwa luas segitiga dengan titik-titik sudut (0, 0), (b, a), (x, y)

adalah|ax− by|

2.

5. Tentukan banyaknya bilangan bulat positif yang dapat dinyatakan dalam

bentuk

4x+ 10y + 19z

dengan x, y, z bilangan bulat non-negatif dan x+ y + z = 94.

64

6. Tentukan bilangan bulat positif a dan b dengan sifat banyaknya bilangan

bulat positif n yang tidak dapat dinyatakan dalam bentuk ax + by untuk

suatu bilangan bulat non-negatif x, y adalah 35 dan salah satu bilangan

tersebut adalah 58.

7. Tunjukkan bahwa170

1991merupakan bilangan rasional terbesar dengan penye-

but sama dengan 1991 yang tidak dapat dinyatakan sebagai jumlahan dua

bilangan rasional dengan penyebut kurang dari 1991.

8. Tentukan bilangan bulat positif terbesar yang tidak dapat dinyatakan dalam

bentuk 42x+ y untuk suatu bilangan bulat positif x dan y dengan y kom-

posit.

9. Diberikan bilangan real positif a, b dan c. Tunjukkan bahwa terdapat paling

sedikit c2/ab pasangan bilangan bulat non-negatif (x, y) yang memenuhi

ax+ by ≤ c.

65

BAB VII

SISTEM NUMERIK DAN FUNGSI TANGGA

7.1 Pendahuluan

Representasi bilangan sangat penting dalam komunikasi matematis, baik di

bidang matematika maupun bidang ilmu lain. Beberapa representasi yang bisa

dijumpai di antaranya biner untuk aljabar Boole, 12-an, dan desimal. Selain itu,

di dunia sains juga dikenal dengan pendekatan atas dan pendekatan bawah untuk

suatu angka real yang kontinum. Sistem pendekatan ini memiliki sifat-sifat yang

unik, yang dapat dipandang sebagai fungsi berundak. Sifat-sifat fungsi berundak

(tangga) merupakan akibat sifat-sifat elementer bilangan bulat.

Untuk itu pada bagian ini dibahas mengenai konsep representasi basis dari

bilangan bulat, beberapa kriteria keterbagian pada representasi desimal, serta

fungsi tangga berupa fungsi floor dan ceilling.

Dengan mempelajari topik bahasan pada Minggu ke-12 dan 13 ini, dihara-

pkan mahasiswa memiliki kemampuan:

1. Mahasiswa mampu menjelaskan fungsi numerik

2. Mahasiswa mampu menjelaskan pengertian fungsi tangga

3. Mahasiswa mampu menerapkan fungsi numerik dan tangga dalam per-

masalahan teori bilangan dan di bidang lain

7.2 Sistem Numerik

Teorema 7.2.1. Diberikan bilangan bulat b > 1. Untuk setiap bilangan bulat

n ≥ 1 , terdapat dengan tunggal sistem bilangan bulat (k, a0, a1, . . . , ak) dengan

sifat 0 ≤ ai ≤ b− 1, i = 0, 1, . . . , k, ak = 0, dan

n = akbk + ak−1b

k−1 + · · ·+ a1b+ a0. (7.5)

Bukti. Pertama-tama, dibuktikan existensi dari sistem tersebut. Berdasarkan

66

Algoritma Pembagian diperoleh

n = q1b+ r1, 0 ≤ r1 ≤ b− 1q1 = q2b+ r1, 0 ≤ r2 ≤ b− 1

. . .qk−1 = qkb+ r1, 0 ≤ rk ≤ b− 1

dengan qk hasil bagi terakhir yang tidak sama dengan nol. Dipilih

q0 = n, a0 = n− q1b, a1 = q1 − q2b, . . . , ak−1 = qk−1 − qkb, ak = qk.

Diperoleh

k∑i=0

aibi =

k−1∑i=0

(qi − qi+1b)bi + qkb

k = q0 +k∑

i=1

aibi −

k∑i=1

aibi = q0 = n.

Selanjutnya, akan dibuktikan ketunggalannya. Diasumsikan n = c0 + c1b+ · · ·+chb

h representasi lain dari n. Akan ditinjau 2 kasus, yaitu h = k dan h = k.

Kasus h = f . Tanpa mengurangi keumuman misalkan h > k, diperoleh n ≥ bh ≥bk+1. Akan tetapi,

n = a0 + a1b+ · · ·+ akbk ≤ (b− 1)(1 + b+ · · ·+ bk = bk+1 − 1 < bk+1,

suatu kontradiksi.

Kasus h = k. Diperhatikan bahwa

a0 + a1b+ · · ·+ akbk = c0 + c1b+ · · ·+ ckb

k,

sehingga diperoleh b|(a0−c0). Di lain pihak, |a0−c0| < b, maka diperoleh a0 = c0.

Akibatnya, didapat

a1 + a2b+ · · ·+ akbk = c1 + c2b+ · · ·+ ckb

k.

Dengan mengulangi prosedur di atas, diperoleh a1 = c1, a2 = b2, . . . , ak = ck. �

Definisi 7.2.2. Diberikan bilangan bulat positif n dan b > 1. Penyajian n ke

dalam bentuk seperti pada (7.5) disebut representasi dari n dalam basis b,

dinotasikan dengan

n = akak−1 . . . a0(b).

67

Khusus untuk b = 10, penyajian tersebut dinamakan representasi

desimal dan cukup dituliskan dalam bentuk n = akak−1 . . . a1a0. (Contoh:

2010 = 2010(10)).

Contoh 7.2.3. Diberikan xy dan yx bilangan bulat positif dua digit. Buktikan

bahwa jumlahan keduanya merupakan bilangan komposit.

Penyelesaian. Karena xy = 10x + y dan yx = 10y + x, maka jumlahan ked-

uanya sama dengan 11x+11y = 11(x+y), yang merupakan bilangan komposit. �

Contoh 7.2.4. Tentukan bilangan bulat yang terdiri dari 6 digit dengan angka

terakhir 7 dan menjadi 5 kali bilangan semula jika digit terakhir dipindahkan

menjadi digit pertama.

Penyelesaian. Misalkan bilangan tersebut abcde7. Diperoleh 7abcde = 5 ×abcde7. Dengan menjabarkan ke dalam representasi desimal diperoleh

7.105 + 104a+ 103b+ 102c+ 10d+ e = 5(105a+ 104b+ 103c+ 102d+ 10e+ 7)

⇔ 490000a+ 49000b+ 4900c+ 490d+ 49e = 699965

⇔ 10000a+ 1000b+ 100c+ 10d+ e = 14285.

Berdasarkan ketunggalan sistem desimal diperoleh a = 1, b = 4, c = 2, d = 8 dan

e = 5. Jadi, bilangan yang dimaksud adalah 142857. �

Contoh 7.2.5. Pada persamaan dibawah ini, masing-masing huruf merepresen-

tasikan secara tunggal suatu digit dalam basis 10:

(Y E) · (ME) = TTT.

Tentukan nilai dari E +M + T + Y .

Penyelesaian. Karena TTT = T ·111 = T ·3 ·37, maka salah satu dari Y E atau

ME adalah 37, yang berakibat E = 37. Karena 0 ≤ T ≤ 9 dan T ·3 bilangan dua

digit dengan digit terakhir 7, maka diperoleh T = 9, dan TTT = 999 = 27 · 37.Jadi, E +M + T + Y = 2 + 3 + 7 + 9 = 21. �

68

Contoh 7.2.6. Tuliskan 1111011(2) ke dalam basis 10 dan tuliskan 2010 ke dalam

basis 7.

Penyelesaian. Diperhatikan bahwa

1111011(2) = 1 · 26 + 1 · 25 + 1 · 24 + 1 · 23 + 0 · 22 + 1 · 21 + 1 · 20

= 64 + 32 + 16 + 8 + 2 + 1 = 123.

Diperoleh representasi desimal dari 1111011(2) adalah 123.

Dengan membagi 2010 dengan 7 secara berulang-ulang, sisanya memberikan

digit-digit dari representasi pada basis 7, dimulai dari yang terakhir. Digit per-

tama adalah hasil bagi terakhir yang tak nol. Susunan perhitungannya diberikan

sebagai berikut.2010 72009 287 71 287 41 7

0 35 56

Jadi, 2010 = 5601(7). �

Contoh 7.2.7. Beberapa himpunan yang terdiri dari bilangan-bilangan prima

seperti {7, 83, 421, 659}, menggunakan setiap digit dari 9 digit tak nol tepat sekali.

Tentukan jumlah terkecil yang mungkin dari anggota-anggota himpunan dengan

kondisi tersebut yang bisa diperoleh.

Penyelesaian. Jawabannya adalah 207. Diperhatikan bahwa digit 4,6 dan 8

tidak bisa muncul pada digit satuan, sehingga diperoleh jumlahan dari anggota-

anggota himpunan dengan kondisi tersebut setidaknya 40 + 60 + 80 + 1 + 2 +

3 + 5 + 7 + 9 = 207. Di lain pihak, nilai ini bisa didapatkan dari himpunan

{2, 5, 7, 43, 61, 89}. �

Contoh 7.2.8. Tentukan semua bilangan bulat positif n sehingga 11111(n) adalah

kuadrat sempurna.

69

Penyelesaian. Diperhatikan bahwa 11111(n) = n4 + n3 + n2 + n+ 1.

Jika n genap, maka n2 + n2dan n2 + n

2+ 1 adalah 2 bilangan bulat berurutan.

Diperoleh (n2 +

n

2

)2= n4 + n3 +

n2

2< n4 + n3 + n2 + n+ 1

<(n2 +

n

2+ 1)2.

Jadi, 11111(n) bukan kuadrat sempurna untuk bilangan genap n.

Jika n ganjil, maka n2 + n2− 1

2dan n2 + n

2+ 1

2adalah bilangan bulat berurutan.

Diperhatikan bahwa(n2 +

n

2− 1

2

)2

< n4 + n3 + n2 + n+ 1.

dan (n2 +

n

2+

1

2

)2

= n4 + n3 +5n2

4+

n

2+

1

2

= n4 + n3 + n2 + n+ 1 +n2 − 2n− 3

4

= n4 + n3 + n2 + n+ 1 +(n− 3)(n+ 1)

4.

Untuk n bilangan ganjil lebih dari 3, 11111(n) berada diantara 2 bilangan bulat

kuadrat berurutan, yaitu(n2 +

n

2− 1

2

)2

dan

(n2 +

n

2+

1

2

)2

.

Jadi, 11111(n) bukan kuadrat sempurna untuk setiap bilangan positif lebih dari

3. Untuk n = 3, diperoleh 11111(3) = 121 = 112. Jadi, bilangan bulat positif

yang memenuhi hanya n = 3. �

Pada contoh terakhir, diperoleh bahwa suatu bilangan bulat bukan

kuadrat sempurna jika bilangan tersebut terletak di antara 2 bilangan kuadrat

berurutan. Cara ini sangat bermanfaat dalam menyelesaikan beberapa masalah

terkait persamaan Diophantine.

Dalam beberapa sistem numerik, basis tidak harus selalu bernilai kon-

stan. Berikut salah satu contohnya.

70

Teorema 7.2.9. Setiap bilangan bulat positif k mempunyai ekspansi basis fak-

torial yang tunggal, katakan

(f1, f2, f3, . . . , fm),

yang berarti bahwa

k = 1! · f1 + 2! · f2 + 3! · f3 + · · ·+m! · fm,

dengan untuk setiap i = 1, 2, . . . ,m, fi bilangan bulat, 0 ≤ fi ≤ i dan fm > 0.

Bukti. Diberikan bilangan bulat positif k. Diperhatikan bahwa terdapat dengan

tunggal bilangan bulat positif m1 dengan sifat m1! ≤ k < (m1+1)!. Berdasarkan

Algoritma Pembagian, diperoleh

k = m1!fm1 + r1

untuk suatu bilangan bulat positif fm1 dan bilangan bulat r1 dengan 0 ≤ r1 <

m1!. Karena k < (m1 +1)! = m1! · (m1 +1), maka fm1 ≤ m. Dengan mengulangi

proses ini, diperoleh

r1 = m2!fm2 + r2,

untuk suatu m2 bilangan bulat positif yang tunggal dengan m2! ≤ r1 < (m2 +

1)!, 1 ≤ fm2 ≤ m2, dan 0 ≤ r2 < m2!. Dengan melanjutkan proses ini berulang-

ulang, akan diperoleh suatu ekspansi basis faktorial yang tunggal dari k. �

Teorema 7.2.10. Diberikan F0 = 1, F1 = 1, dan Fn+1 = Fn + Fn−1 untuk setiap

bilangan bulat positif n. (Barisan ini dinamakan barisan Fibonacci, dan suku-

sukunya dinamakan bilangan Fibonacci. Setiap bilangan bulat non-negatif n

dapat dituliskan dengan tunggal sebagai suatu jumlahan dari bilangan-bilangan

Fibonacci yang tidak berurutan, yaitu

n =∞∑k=0

αkFk,

dengan αk ∈ {0, 1} dan (αk, αk +1) = (1, 1) untuk setiap k. Ekspresi untuk n ini

disebut dengan representasi Zeckendorf dari n.

Bukti dari teorema ini hampir sama dengan teorema sebelumnya.

71

Contoh 7.2.11. Diberikan (f1, f2, f3, . . . , fj) adalah ekspansi basis faktorial dari

16!− 32! + 48!− 64! + · · ·+ 1968!− 1984! + 2000!.

Tentukan nilai dari f1 − f2 + f3 − f4 + · · ·+ (−1)j−1fj.

Penyelesaian. Karena (n+ 1)!− n! = n!(n+ 1)− n! = n!n, maka diperoleh

(n+ 16)!− n! = (n+ 16)!− (n+ 15)! + (n+ 15)!− (n+ 14)! + · · ·+ (n+ 1)!− (n)!

= (n+ 15)!(n+ 15) + (n+ 14)!(n+ 14) + · · ·+ (n+ 1)!(n+ 1) + n!n.

Akibatnya, diperoleh ekspansi basis faktorial dari (n+ 16)!− n! adalah

(0, 0, . . . , 0, n, n+ 1, . . . , n+ 14, n+ 15),

dengan banyak bilangan nol pada bagian awal adalah n − 1. Diperhatikan

bahwa ekspansi basis faktorial dari 16! adalah (0, 0, . . . , 0, 1), sehingga diperoleh

ekspansi yang dicari adalah

(0, 0, . . . , 0; 1; 0, . . . , 0; 32, 33, . . . , 47;

0, . . . , 0; 64, . . . , 79; . . . ; 1984, . . . , 1999).

Diperhatikan bahwa dimulai pada posisi 32,ekspansi tersebut terdiri dari

kelompok atas 16 bilangan tak nol yang saling bergantian dengan kelompok atas

16 bilangan nol. Dengan pengecualian untuk f16 = 1, nilai dari setiap fi yang

tak kosong adalah i. Masing-masing dari 62 kelompok atas 16 bilangan tak nol

memberikan hasil 8 untuk jumlahan yang dicari, dan f16 memberikan hasil -1.

Jadi, nilai dari jumlahan yang dicari adalah 8 · 62− 1 = 495. �

7.3 Kriteria Keterbagian pada Sistem Desimal

Diberikan beberapa kriteria keterbagian untuk bilangan bulat pada repre-

sentasi desimal.

Teorema 7.3.1. Diberikan n = ahah−1 . . . a0 bilangan bulat positif.

72

a. Jika S(n) = a0+a1+ . . .+ah merupakan jumlah digit-digit dari n, maka

n ≡ s(n) (mod 3). Secara khusus, n habis dibagi oleh 3 jika dan hanya

jika S(n) habis dibagi 3.

b. Jika S(n) = a0+a1+ . . .+ah merupakan jumlah digit-digit dari n, maka

n ≡ S(n) (mod 9). Secara khusus, n habis dibagi oleh 9 jika dan hanya

jika S(n) habis dibagi 9.

c. Jika s′(n) = a0 − a1 + . . . + (−1)hah, maka n habis dibagi oleh 11 jika

dan hanya jika s(n) habis dibagi 11.

d. n habis dibagi oleh 7,11 atau 13 jika dan hanya jika ahah−1 . . . a3−a2a1a0

habis dibagi oleh bilangan yang sama.

e. n habis dibagi oleh 27 atau 37 jika dan hanya jika ahah−1 . . . a3 + a2a1a0

habis dibagi oleh bilangan yang sama.

f. n berturut-turut habis dibagi oleh 2k atau 5k dengan k ≤ h jika dan hanya

jika ahah−1 . . . a0 habis dibagi oleh bilangan yang sama.

Contoh 7.3.2. Diberikan a679b bilangan lima digit yang habis dibagi 72. Ten-

tukan nilai a dan b.

Penyelesaian. Diperhatikan bahwa 72 = 8.9. Karena 8 dan 9 relatif prima,

maka a679b habis dibagi oleh 8 dan 9. Karena a679b habis dibagi 8, maka 79b

habis dibagi 8. Diperoleh nilai b yang memenuhi hanya b = 2. Karena a6792

habis dibagi 9, maka a+ 6+ 7+ 9+ 2 habis dibagi 9. Diperoleh bilangan a yang

memenuhi hanya a = 3. �

Contoh 7.3.3. Kuadrat sempurna atau tidak?

a. Tentukan semua bilangan bulat positif k dengan sifat bilangan k-digit 11 . . . 1

kuadrat sempurna.

b. Apakah terdapat bilangan kuadrat sempurna 5 digit yang terdiri dari digit-digit

genap berbeda.

c. Tentukan semua bilangan bulat positif n dengan sifat bilangan n digit 200 . . . 013

merupakan bilangan kuadrat sempurna.

73

Penyelesaian.

a. Jelas k = 1 memenuhi. Diperhatikan bahwa untuk k ≥ 1 berlaku

11 . . . 1︸ ︷︷ ︸k kali

≡ 11 ≡ 3 (mod 4),

yang berarti 11 . . . 1 bukan kuadrat sempurna.

b. Tidak. Diperhatikan bahwa 0 + 2 + 4 + 6 + 8 = 20 ≡ 2 (mod 9), sedangkan

setiap bilangan kuadrat sempurna kongruen 0, 1, 4, 7 modulo 9. Akibatnya

setiap bilangan kuadrat sempurna 5 digit yang terdiri dari digit-digit genap

berbeda bukan kuadrat sempurna.

c. Diperhatikan bahwa untuk setiap bilangan bulat positif n, jumlah digit-digit

bilangan tersebut sama dengan 6, kelipatan 3 tapi bukan kelipatan 9. Jadi,

tidak ada bilangan bulat positif n dengan sifat bilangan n digit 200 . . . 013

merupakan bilangan kuadrat sempurna.

Contoh 7.3.4. Tentukan banyaknya bilangan 5 digit abcde dengan sifat abc+ de

habis dibagi 11.

Penyelesaian. Diperhatikan bahwa

abcde = abc× 100 + de = (abc+ de) + 99× abc.

Diperoleh abc + de habis dibagi 11 jika dan hanya jika abcde habis dibagi 11.

Diperhatikan bahwa bilangan 5 digit terbesar yang habis dibagi oleh 11 adalah

99990 (9090× 11) dan bilangan 4 digit terbesar yang habis dibagi oleh 11 adalah

9999 (909 × 11). Diperoleh ada tepat 9090 − 909 = 8181 bilangan 5 digit yang

merupakan kelipatan 11. Jadi, banyaknya ada 8181 bilangan. �

Teorema 7.3.5. Diberikan n1, n2 bilangan bulat positif. Jika S(n) menyatakan

jumlah digit-digit dari n, maka

a. 9|S(n1)− n1;

74

b. S(n1 + n2) ≤ S(n1) + S(n2);

c. S(n1n2) ≤ min(n1S(n2), n2S(n1));

d. S(n1n2) ≤ S(n1)S(n2).

Bukti. Misalkan n1 = akak−1 . . . a0, n2 = bhbh−1 . . . b0 dan n1+n2 = cscs−1 . . . c0.

a. Cukup jelas dari Teorema 7.3.1.

b. Dipilih t terkecil dengan sifat ai + bi < 10 untuk setiap i < t. Diperoleh

at + bt ≥ 10, sehingga ct = at + bt − 10 dan ct+1 = at+1 + bt+1 + 1. Akibatnya

berlakut+1∑i=1

ci ≤t+1∑i=1

ai +t+1∑i=1

bi.

Dengan melanjutkan proses ini diperoleh S(n1 + n2) ≤ S(n1) + S(n2).

c. Tanpa mengurangi keumuman misalkan min(n1S(n2), n2S(n1)) = n1S(n2).

Dengan menggunakan bagian b., diperoleh

S(n1n2) = S (n2 + n2 + . . .+ n2)︸ ︷︷ ︸n1 kali

≤ S(n2) + S(n2) + . . .+ S(n2)︸ ︷︷ ︸n1 kali

= n1S(n2).

d. Diperhatikan bahwa dari bagian b. dan c. berlaku

S(n1n2) = S

(n1

h∑i=0

bi10i

)= S

(h∑

i=0

n1bi10i

)

≤h∑

i=0

S(n1bi10

i)=

h∑i=0

S (n1bi)

≤h∑

i=0

biS (n1) = S (n1)h∑

i=0

bi

= S (n1)S (n2) .

75

Contoh 7.3.6. Tentukan bilangan bulat positif n dengan sifat S(n) = 2013S(3n).

Penyelesaian. Diambil

n = 133 . . . 3︸ ︷︷ ︸6037 kali

5,

diperoleh

n = 400 . . . 0︸ ︷︷ ︸6037 kali

5.

Diperhatikan bahwa S(n) = 3.6037 + 1 + 5 = 18117 = 2013.9 = 2013S(3n). �

Contoh 7.3.7. Misalkan jumlahan digit-digit dari representasi desimal 44444444

adalah A. Jika B adalah jumlahan digit-digit dari A, maka tentukan jumlahan

digit-digit dari B.

Penyelesaian. Misalkan a = 44444444. Diperhatikan bahwa 4444 < 104, maka

diperoleh

a = 44444444 < 104.4444 = 1017776,

sehingga banyaknya digit dari a tidak lebih dari 17776 digit. Karena setiap

digit kurang dari atau sama dengan 9, maka diperoleh A = S(a) ≤ 17776.9 =

159984. Diantara bilangan bulat positif yang kurang dari atau sama dengan

159984, bilangan dengan jumlah digit terbesar adalah 99999, sehingga diperoleh

B = S(A) ≤ 45. Diantara bilangan bulat positif yang kurang dari atau sama

dengan 45, bilangan dengan jumlah digit terbesar adalah 39, sehingga diperoleh

S(B) ≤ 12. Diperhatikan bahwa

S(B) ≡ B ≡ S(A) ≡ A ≡ S(a) ≡ a ≡ 44444444 (mod 9)

dan

44444444 ≡ (4 + 4 + 4 + 4)4444 ≡ 164444 ≡ (−2)4444

≡ (−2)3.1481+1 ≡ (−8)1481.(−2)

≡ 1.(−2) ≡ 7 (mod 9).

Diperoleh bahwa nilai S(B) adalah 7. �

Soal Latihan

76

1. Tentukan jumlah semua bilangan 2 digit yang habis dibagi oleh setiap dig-

itnya.

2. Tentukan bilangan bulat positif terkecil n dengan sifat representasi desimal

dari 15n hanya memuat digit 0 dan 8.

3. Tentukan bilangan 6 digit abcdef yang memenuhi 7×abcdef = 6×defabc.

4. Tentukan bilangan bulat positif terkecil yang merupakan kelipatan 84 dan

hanya terdiri dari digit 6 atau 7.

5. Sebuah speedometer yang rusak pada sebuah mobil memproses dari digit

3 ke digit 5, selalu melewati digit 4, tanpa menghiraukan letak kesalahan.

Contohnya, setelah melakukan perjalanan 1 km, speedometer berubah dari

000039 ke 000050. Jika pada odometer tersebut sekarang terbaca 002005,

sudah berapa km jarak yang ditempuh mobil tersebut dari awal?

6. Diketahui n bilangan 7 digit yang habis dibagi oleh setiap digitnya. Ten-

tukan tiga digit yang bukan digit dari n.

7. Diketahui M = 11 . . . 1︸ ︷︷ ︸62 kali

dan N = 11 . . . 1︸ ︷︷ ︸61 kali

. Tentukan jumlah digit-digit dari

3MN .

8. Diberikan bilangan 7 digit M = abcdefg. Misalkan N = gfedcba. Tun-

jukkan representasi desimal dari M + N memuat setidaknya satu digit

genap.

9. Untuk setiap bilangan bulat positif n, didefinisikan p(n) sebagai hasil kali

digit-digit tak nol dari n. Misalkan

S = p(1) + p(2) + . . .+ p(999).

Tentukan faktor prima terbesar dari S.

10. Tunjukkan bahwa setiap bilangan bulat positif memiliki kelipatan yang

memuat digit-digit 0, 1, 2, . . . , 9.

11. Tentukan bilangan bulat positif terkecil x dengan sifat tiga digit terakhir

dari x3 adalah 888.

77

12. Tunjukkan bahwa bilangan 11 . . . 19 dalam basis 9 merupakan jumlahan dari

k bilangan bulat positif pertama untuk suatu bilangan bulat positif k.

13. Diberikan bilangan bulat positif n dengan sifat setiap digitnya (kecuali digit

pertama) lebih besar dari digit di sebelah kirinya. Tentukan jumlah digit-

digit dari 9n.

14. Tunjukkan bahwa untuk setiap bilangan bulat positif n, terdapat bilangan

n digit yang habis dibagi 5n dan semua digitnya bilangan ganjil.

7.4 Fungsi Tangga

Diperhatikan bahwa untuk setiap bilangan real x terdapat dengan tunggal

bilangan bulat n dengan sifat n ≤ x < n+ 1.

Definisi 7.4.1. Diberikan bilangan real x. Bilangan bulat yang lebih besar dari

atau sama dengan x disebut floor dari x, ditulis n = ⌊x⌋. Selisih x−⌊x⌋ disebut

bagian pecahan dari x, dinotasikan dengan {x}. Bilangan bulat yang lebih besar

dari atau sama dengan x disebut ceiling dari x, dinotasikan dengan n = ⌈x⌉.

Diperhatikan bahwa jika x bilangan bulat, maka ⌊x⌋ = ⌈x⌉ dan {x} = 0,

sedangkan jika x bukan bilangan bulat, maka ⌈x⌉ = ⌊x⌋+ 1.

Contoh 7.4.2. Selesaikan sistem persamaan berikut:

x+ ⌊y⌋+ {z} = 200.0,

{x}+ y + ⌊z⌋ = 190.1,

⌊x⌋+ {y}+ z = 178.8.

Penyelesaian. Karena untuk setiap bilangan real x berlaku x = ⌊x⌋+{x}, maka

dengan menjumlahkan ketiga persamaan tersebut diperoleh

2x+ 2y + 2z = 568.9, or x+ y + z = 284.45.

Dengan mengurangkan masing-masing persamaan yang diberikan dengan per-

samaan terakhir diperoleh

{y}+ ⌊z⌋ = 84.45,

⌊x⌋+ {z} = 94.35,

{x}+ ⌊y⌋ = 105.65.

78

Didapat {y} = 0.45, ⌊z⌋ = 84, ⌊x⌋ = 94, {z} = 0.35, {x} = 0.65, ⌊y⌋ = 105. Jadi,

x = 94.65, y = 105.45 dan z = 84.35. �

Contoh 7.4.3. Diberikan bilangan real positif a dengan {a−1} = {a2} dan 2 <

a2 < 3. Tentukan nilai dari a6 − 8a−1.

Penyelesaian. Diperhatikan bahwa a > 1, maka a−1 < 1, sehingga diperoleh

{a−1} = a−1. Karena 2 < a2 < 3, maka {a2} = a2 − 2. Akibatnya diperoleh

a−1 = {a−1} = {a2} = a2 − 2 atau a3 − 2a− 1 = 0. Diperoleh

(a+ 1)(a2 − a− 1) = a3 − 2a− 1 = 0.

Nilai a positif yang memenuhi hanya a =1 +

√5

2. Diperhatikan bahwa

a6 = (a3)2 = (2a+ 1)2 = 4a2 + 4a+ 1 = 4(a+ 1) + 4a+ 1 = 8a+ 5.

Diperoleh a7 = 8a2 + 5a = 13a+ 8, sehingga berlaku

a6 − 8a−1 =a7 − 8

a= 13.

Contoh 7.4.4. Tentukan semua solusi real dari persamaan

4x2 − 40 ⌊x⌋+ 51 = 0.

Penyelesaian. Diperhatikan bahwa karena untuk setiap bilangan real x berlaku

⌊x⌋ ≤ x, maka diperoleh

(2x− 3)(2x− 17) = 4x2 − 40x+ 51 ≤ 4x2 − 40 ⌊x⌋+ 51 = 0,

sehingga didapat 32≤ x ≤ 17

2. Akibatnya diperoleh 1 ≤ ⌊x⌋ ≤ 8. Di lain pihak

x =

√40 ⌊x⌋ − 51

2,

sehingga berlaku

⌊x⌋ =

⌊√40 ⌊x⌋ − 51

2

⌋.

79

Dengan mensubstitusi ⌊x⌋ ∈ {1, 2, 3, . . . , 8} ke persamaan tesebut, diperoleh

bahwa nilai ⌊x⌋ yang memenuhi hanya 2, 6, 7 atau 8. Diperoleh nilai x yang

memenuhi adalah√292,√1892

,√2292

dan√2692

. �

Contoh 7.4.5. Tentukan bilangan bulat terbesar yang kurang dari atau sama

dengan (√6 +

√5)4.

Penyelesaian. Dengan menggunakan Binomial Newton diperoleh

(√6 +

√5)4 = (

√6)4 + 4(

√6)3

√5 + 6(

√6)2(

√5)2 + 4

√6(√5)3 + (

√5)4

= 36 + 24√30 + 180 + 20

√30 + 25 = 241 + 49

√30

dan

(√6−

√5)4 = (

√6)4 − 4(

√6)3

√5 + 6(

√6)2(

√5)2 − 4

√6(√5)3 + (

√5)4

= 36− 24√30 + 180− 20

√30 + 25 = 241− 49

√30.

Diperoleh

(√6 +

√5)4 + (

√6−

√5)4 = 482.

Diperhatikan bahwa√6−

√5 < 1, maka (

√6−

√5)4 < 1. Akibatnya diperoleh⌊

(√6 +

√5)4⌋= 481.

Beberapa karakteristik dari fungsi floor atau fungsi tangga diberikan

sebagai berikut.

Teorema 7.4.6. Diberikan bilangan real x, y.

a. Jika a dan b bilangan bulat dengan b > 0 dan q, r berturut-turut hasil

bagi dan sisa ketika a dibagi oleh b, maka q =⌊ab

⌋dan r =

{ab

}.b.

b. Untuk setiap bilangan bulat n berlaku ⌊x+ n⌋ = ⌊x⌋ + n dan ⌈x+ n⌉ =⌈x⌉+ n.

c. Jika x bilangan bulat, maka ⌊x⌋+⌊−x⌋ = 0; jika x bukan bilangan bulat,

maka ⌊x⌋+ ⌊−x⌋ = 1.

80

d. Jika x ≤ y, maka ⌊x⌋ ≤ ⌊y⌋.

e. ⌊x⌋+ ⌊y⌋ ≤ ⌊x+ y⌋ ≤ ⌊x⌋+ ⌊y⌋+ 1.

f. Jika x, y ≥ 0, maka ⌊x⌋ · ⌊y⌋ ≤ ⌊xy⌋.

g. Jika x ≥ 0, maka untuk setiap bilangan bulat positif n banyaknya keli-

patan positif dari n yang kurang dari atau sama dengan x adalah⌊xn

⌋.

h. Untuk setiap bilangan bulat positif n berlaku⌊⌊x⌋n

⌋=⌊xn

⌋.

Contoh 7.4.7. Tunjukkan bahwa untuk setiap bilangan real x dan y berlaku

⌊2x⌋+ ⌊2y⌋ ≥ ⌊x⌋+ ⌊y⌋+ ⌊x+ y⌋ .

Penyelesaian. Diberikan bilangan real x dan y. Karena x = ⌊x⌋ + {x} dan

y = ⌊y⌋+ {y}, maka diperoleh

⌊2x⌋+ ⌊2y⌋ = 2 ⌊x⌋+ ⌊2{x}⌋+ 2 ⌊y⌋+ ⌊2{y}⌋

dan

⌊x+ y⌋ = ⌊x⌋+ ⌊y⌋+ ⌊{x}+ {y}⌋ .

Akibatnya, cukup dibuktikan

⌊2{x}⌋+ ⌊2{y}⌋ ≥ ⌊{x}+ {y}⌋ .

Tanpa mengurangi keumuman, misalkan {x} ≥ {y}. Diperoleh 2{x} ≥ {x}+{y},sehingga diperoleh

⌊2{x}⌋+ ⌊2{y}⌋ ≥ ⌊2{x}⌋ ⌊{x}+ {y}⌋ .

Jadi,

⌊2x⌋+ ⌊2y⌋ ≥ ⌊x⌋+ ⌊y⌋+ ⌊x+ y⌋ .

81

Contoh 7.4.8. Tunjukkan bahwa untuk setiap bilangan bulat positif n berlaku⌊√n+

√n+ 1

⌋=⌊√

4n+ 2⌋.

Penyelesaian. Diambil sebarang bilangan bulat positif n. Diperhatikan bahwa

(√n+

√n+ 1)2 = 2n+ 1 + 2

√n2 + n

dan

n <√n2 + n < n+ 1.

Diperoleh √4k + 1 <

√n+

√n+ 1 <

√4n+ 3.

Karena setiap bilangan kuadrat bersisa 0 atau 1 ketika dibagi oleh 4, maka 4n+2

dan 4n+ 3 bukan bilangan kuadrat, sehinga diperoleh⌊√4k + 1

⌋=⌊√

4k + 2⌋=⌊√

4k + 3⌋.

Jadi, ⌊√n+

√n+ 1

⌋=⌊√

4n+ 2⌋.

Contoh 7.4.9. Diberikan p dan q bilangan bulat yang relatif prima. Tunjukkan

bahwa ⌊p

q

⌋+

⌊2p

q

⌋+ . . .+

⌊(q − 1)p

q

⌋=

(p− 1)(q − 1)

2.

Penyelesaian. Karena gcd(p, q) = 1, makaip

qbukan bilangan bulat untuk setiap

i = 1, 2, . . . , q − 1. Diperoleh⌊ip

q

⌋+

⌊(q − i)p

q

⌋= p+

⌊ip

q

⌋+

⌊−ip

q

⌋= p− 1

untuk setiap i = 1, 2, . . . , q − 1. Akibatnya,

2

q−1∑i=1

⌊ip

q

⌋=

q−1∑i=1

(⌊ip

q

⌋+

⌊(q − i)p

q

⌋)

=

q−1∑i=1

(p− 1) = (p− 1)(q − 1).

82

Jadi, ⌊p

q

⌋+

⌊2p

q

⌋+ . . .+

⌊(q − 1)p

q

⌋=

(p− 1)(q − 1)

2.

Selanjutnya, diberikan salah satu sifat terkenal terkait fungsi tangga

yaitu Identitas Hermit.

Teorema 7.4.10 (Identitas Hermit). Untuk setiap bilangan real x dan bilangan

bulat positif n berlaku

⌊x⌋+⌊x+

1

n

⌋+

⌊x+

2

n

⌋+ . . .+

⌊x+

n− 1

n

⌋= ⌊nx⌋ .

Bukti. Diambil sebarang bilangan real x dan bilangan bulat positif n. Kasus

x bilangan bulat cukup jelas. Diasumsikan x tidak bulat, berarti 0 < {x} < 1,

sehingga terdapat 1 ≤ i ≤ n− 1 dengan sifat

{x}+ i− 1

n< n dan {x}+ i

n≥ 1,

ekuivalen dengann− i

n≤ {x} <

n− i+ 1

n.

Akibatnya diperoleh

⌊x⌋ =⌊x+

1

n

⌋= . . . =

⌊x+

i− 1

n

⌋dan ⌊

x+i

n

⌋=

⌊x+

i+ 1

n

⌋= . . . =

⌊x+

n− 1

n

⌋,

sehingga berlaku

⌊x⌋+⌊x+

1

n

⌋+ . . .+

⌊x+

n− 1

n

⌋= i ⌊x⌋+ (n− i)(⌊x⌋+ 1)

= n ⌊x⌋+ n− i.

Di sisi lain,

n ⌊x⌋+ n− i ≤ n ⌊x⌋+ n{x} = nx < n ⌊x⌋+ n− i+ 1,

83

yang berarti ⌊nx⌋ = n ⌊x⌋+ n− i. Jadi,

⌊x⌋+⌊x+

1

n

⌋+ . . .+

⌊x+

n− 1

n

⌋= n ⌊x⌋+ n− i

= ⌊nx⌋ .

Contoh 7.4.11. Diberika bilangan real x. Tunjukkan bahwa

∞∑k=1

⌊x+ 2k

2k+1

⌋= ⌊x⌋ .

Penyelesaian. Dengan mengambil n = 2 pada Identitas Hertmit diperoleh

⌊x⌋+⌊x+

1

2

⌋= ⌊2x⌋ ,

atau ⌊x+

1

2

⌋= ⌊2x⌋ − ⌊x⌋ .

Akibatnya diperoleh

∞∑k=1

⌊x+ 2k

2k+1

⌋=

∞∑k=1

⌊x

2k+1+

1

2

⌋=

∞∑k=1

(⌊ x2k

⌋−⌊ x

2k+1

⌋)= ⌊x⌋ .

7.5 Pangkat Tertinggi

Fungsi floor memiliki cukup banyak aplikasi, salah satunya dalam menen-

tukan pangkat tertinggi suatu bilangan pada faktorisasi prima bilangan berbentuk

n!.

Teorema 7.5.1. Diberikan bilangan bulat positif n dan p dengan p prima. Bi-

langan bulat non-negatif k yang memenuhi pk∥n! adalah⌊n

p

⌋+

⌊n

p2

⌋+

⌊n

p3

⌋+ . . . .

84

Bukti. Akan dibuktikan dengan induksi matematika.

Basis Induksi. Untuk n = 1 diperoleh p0∥1!. Di sisi lain, untuk setiap bilangan

bulat positif s berlaku ⌊1

ps

⌋= 0,

sehingga diperoleh

0 =

⌊1

p

⌋+

⌊1

p2

⌋+

⌊1

p3

⌋+ . . . .

Terbukti untuk kasus n = 1.

Langkah Induksi. Diasumsikan pernyataan benar untuk setiap n < m.

Akan ditunjukkan pernyataan benar untuk n = m. Misalkan k memenuhi pk∥m!.

Diperhatikan bahwa banyaknya bilangan kelipatan p di antara 1, 2, . . . ,m adalah⌊m

p

⌋. Diperoleh k memenuhi

pk∥p(2p)(3p) . . . (k1p) = pk1(k1)!,

dengan k1 =

⌊m

p

⌋, sehingga cukup dicari k dengan sifat

pk−k1∥(k1)!

Berdasarkan asumsi induksi diperoleh

k − k1 =

⌊k1p

⌋+

⌊k1p2

⌋+

⌊k1p3

⌋+ . . . .

Karena untuk setiap bilangan bulat positif s berlaku

⌊k1ps

⌋=

⌊m

p

⌋ps

=

⌊m

ps

⌋,

maka diperoleh

k −⌊m

p

⌋=

⌊m

p2

⌋+

⌊m

p3

⌋+

⌊m

p4

⌋+ . . . .

Jadi,

k =

⌊m

p

⌋+

⌊m

p2

⌋+

⌊m

p3

⌋+ . . . .

Terbukti pernyataan benar untuk kasus n = m. �

85

Contoh 7.5.2. Tentukan banyaknya digit nol berurutan yang terletak pada bagian

akhir dari representasi desimal 2013!.

Penyelesaian. Akan dicari m dengan sifat 10m∥2013!. Diperhatikan bahwa

10 = 2.5 dan 2 < 5. Diperoleh bilangan p, q dengan 2p∥2013! dan 5q∥2013!memenuhi q < p, sehingga didapat m = q. Akibatnya

m = q =

⌊2013

5

⌋+

⌊2013

25

⌋+

⌊2013

125

⌋+

⌊2013

625

⌋= 402 + 80 + 16 + 3 = 501.

Contoh 7.5.3. Diberikan m dan n bilangan bulat positif. Tunjukkan bahwa

m!(n!)m habis membagi (mn)!.

Penyelesaian. Diambil sebarang bilangan prima p. Misalkan w, x, y, z bilangan

bulat non-negatif dengan sifat pw∥m!, px∥n!, py∥m!(n!)m dan pz∥(mn)!. Cukup

ditunjukkan x ≤ y. Diperhatikan bahwa y = w+mx, sehingga cukup ditunjukkan

∞∑i=1

⌊mn

pi

⌋≥

∞∑i=1

⌊m

pi

⌋+m

∞∑i=1

⌊n

pi

⌋.

Jelas bahwa jika p > n, jumlahan kedua pada ruas kanan bernilai 0, sehingga

ketaksamaan benar. Diasumsikan p ≤ n. Misalkan s bilangan bulat positif

dengan sifat ps ≤ n < ps+1. Diperoleh

∞∑i=1

⌊mn

pi

⌋=

s∑i=1

⌊m

n

pi

⌋+

∞∑i=1

⌊m

pin

ps

⌋≥ m

s∑i=1

⌊n

pi

⌋+

∞∑i=1

⌊m

pi

⌋⌊n

ps

⌋≥ m

∞∑i=1

⌊n

pi

⌋+

∞∑i=1

⌊m

pi

⌋.

Soal Latihan

86

1. Diketahui x dan y bilangan real dengan sifat ⌊√x⌋ = 9 dan

⌊√y⌋= 12.

Tentukan nilai terkecil yang mungkin dicapai oleh ⌊y − x⌋.

2. Diketahui n bilangan bulat positif dengan 7 digit terakhir dari n! adalah

8000000. Tentukan nilai n.

3. Diketahui s dan t bilangan bulat positif dengan sifat

7s∥400! dan 3t∥((3!)!)!.

Tentukan nilai s+ t.

4. Tentukan bilangan bulat positif terkecil n dengan sifat 10290|n!.

5. Tentukan semua bilangan real x yang memenuhi⌊3

x

⌋+

⌊4

x

⌋= 5.

6. Tentukan sisa pembagian

⌊1066

1033 + 3

⌋oleh 1000.

7. Tunjukkan bahwa untuk setiap bilangan bulat positif n berlaku⌊√n+

√n+ 1

⌋=⌊√

n+√n+ 2

⌋.

8. Tunjukkan bahwa untuk setiap bilangan bulat positif n berlaku⌊√n+

1

2

⌋=

⌊√n− 3

4+

1

2

⌋.

9. Tunjukkan bahwa untuk setiap bilangan bulat positif m dan n berlaku

(a) m!n!(m+ n)! membagi (2m)!(2n)!.

(b) (k!)kn+kn−1+...+k+1|(kn+1)!.

10. Diberikan bilangan real r dengan sifat⌊r +

19

100

⌋+

⌊r +

20

100

⌋+ . . .+

⌊r +

91

100

⌋= 546.

Tentukan ⌊100r⌋.

87

11. Tentukan banyaknya bilangan bulat berbeda pada barisan⌊12

2005

⌋,

⌊22

2005

⌋, . . . ,

⌊20052

2005

⌋.

12. Diberikan q =

√5 + 1

2dan n bilangan bulat positif. Tentukan nilai dari

⌊q ⌊qn⌋⌋ − ⌊q2n⌋.

13. Tunjukkan bahwa untuk setiap bilangan bulat positif n,⌊(2 +

√3)n⌋

merupakan bilangan ganjil.

14. Diberikan p dan q bilangan bulat positif. Tunjukkan bahwa

q−1∑i=1

⌊ip

q

⌋=

(q − 1)(p− 1)

2+

gcd(p, q)− 1

2.

15. Tunjukkan bahwa untuk setiap bilangan prima p > 2 berlaku⌊(2 +

√5)p⌋− 2p+1

habis dibagi oleh p.

88

Soal-Soal Tambahan

1. (a) Diberikan n bilangan bulat lebih dari 2. Tunjukkan bahwa diantara

bilangan-bilangan1

n,2

n, . . . ,

n− 1

n,

ada sebanyak genap bilangan yang tidak dapat disederhanakan.

(b) Tunjukkan bahwa12n+ 1

30n+ 2

tidak dapat disederhanakan untuk setiap bilangan bulat positif n.

2. Diberikan k bilangan bulat positif lebih dari 1. Tunjukkan bahwa terdapat

bilangan prima p dan barisan bilangan bulat positif a1, a2, . . . , an dengan

ai < aj untuk setiap i < j dan setiap suku pada barisan

p+ ka1, p+ ka2, p+ ka3 . . .

merupakan bilangan prima.

3. Diberikan bilangan bulat positif m dan n dengan sifat

lcm(m,n) + gcd(m,n) = m+ n.

Tunjukkan bahwa salah satu diantara dua bilangan tersebut habis dibagi

oleh bilangan yang lain.

4. Tunjukkan bahwa untuk setiap bilangan bulat positif a dan b berlaku

(36a+ b)(a+ 36b) = 2k

untuk setiap bilangan bulat positif k.

89

5. Tentukan jumlah semua bilangan berbentuk a/b dengan a dan b merupakan

faktor positif dari 27000 yang relatif prima.

6. Diberikan x, y, z bilangan bulat positif dengan sifat

1

x− 1

y=

1

z.

Misalkan h = gcd(x, y, z). Tunjukkan bahwa hxyz dan h(y− z) merupakan

bilangan kuadrat sempurna.

7. Diberikan p bilangan prima dengan p ≡ 2 (mod 3) dan p|a2+ab+ b2 untuk

suatu bilangan bulat a dan b. Tunjukkan bahwa a dan b habis dibagi oleh

p.

8. Tentukan semua bilangan bulat positif n dengan sifat 3√n! + 5 bilangan

bulat.

9. Tentukan semua bilangan prima p dengan sifat τ(p2 + 11) = 6.

10. Diberikan bilangan bulat positif n. Jika a ≡ b (mod n), tunjukkan bahwa

an ≡ bn (mod n2). Apakah sebaliknya tetap berlaku?

11. Diberikan p bilangan prima dan k bilangan bulat dengan 1 ≤ k ≤ p − 1.

Tunjukkan bahwa (p− 1

k

)≡ (−1)k (mod p).

12. Diberikan bilangan prima p. Tunjukkan bahwa terdapat tak hingga banyaknya

bilangan bulat positif n dengan sifat p|2n − n.

13. Diberikan k bilangan ganjil positif. Tunjukkan bahwa

1 + 2 + . . .+ n|(1k + 2k + . . .+ nk)

untuk setiap bilangan bulat positif n.

14. Diberikan p bilangan prima lebih dari 5. Tunjukkan bahwa tidak ada bi-

langan bulat x dengan sifat x4 = p− 4.

15. Tunjukkan bahwa untuk setiap bilangan bulat positif n berlaku

σ(1) + σ(2) + . . .+ σ(n) ≤ n2.

90

16. Tentuka semua himpunan berhingga atas bilangan bulat positif yang memenuhi

i+ j

gcd(i+ j)

merupakan anggota S untuk setiap i, j ∈ S.

17. Diketahui bahwa 229 merupakan bilangan sembilan digit dengan setiap dig-

itnya berbeda. Tentukan digit diantara 0,1,2,. . . ,9 yang bukan merupakan

digit dari 229.

18. Tunjukkan bahwa untuk setiap bilangan bulat lebih dari 1, bilangan n5 +

n4 + 1 merupakan bilangan komposit.

19. Bilangan 10 digit dikatakan ”menarik” jika setiap digitnya berbeda dan

merupakan kelipatan dari 11111. Tentukan banyaknya bilangan menarik.

20. Tentukan semua bilangan prima p dan q dengan sifat pq|(5p − 2p)(5q − 2q).

21. Tunjukkan bahwa terdapat tak hingga banyaknya bilangan yang tidak memuat

digit 0 dan habis dibagi oleh jumlah dari digit-digitnya.

22. Diberikan a dan b bilangan bulat positif yang relatif prima dan dibentuk

deret artimatik berikut: a, a+ b, a+ 2b, . . ..

(a) Tunjukkan bahwa terdapat tak hingga banyaknya suku dari barisan

aritmatik tersebut yang memiliki faktor prima sama.

(b) Tunjukkan bahwa terdapat tak hingga banyaknya pasangan suku dari

barisan aritmatik tersebut yang relatif prima.

23. Diberikan n bilangan bulat positif.

(a) Tentukan gcd(n! + 1, (n+ 1)! + 1).

(b) Diberikan a dan b bilangan bulat positif. Tunjukkan bahwa

gcd(na − 1, nb − 1) = ngcd(a,b) − 1.

(c) Diberikan a dan b bilangan bulat positif. Tunjukkan bahwa gcd(na +

1, nb + 1) habis membagi ngcd(a,b) + 1.

91

(d) Diberikan m bilangan bulat positif dengan gcd(m,n) = 1. Tentukan

gcd(5m + 7m, 5n + 7n).

24. Diberikan a bilangan bulat dengan sifat

1 +1

2+

1

3+ . . .+

1

23=

a

23!.

Tentukan sisa pembagian a oleh 13.

25. (a) Diberikan p > 3 bilangan prima dan m,n bilangan bulat yang relatif

prima dengan sifat

m

n=

1

12+

1

22+ . . .+

1

(p− 1)2.

Tunjukkan bahwa p|m.

(b) Diberikan p > 3 bilangan prima. Tunjukkan bahwa

p2|(p− 1)!

(1 +

1

2+ . . .+

1

p− 1

).

26. Tentukan semua pasangan bilangan bulat non-negatif (x, y) dengan sifat

x2 + 3y dan y2 + 3x merupakan bilangan kuadrat sempurna.

27. Tentukan tiga digit terakhir dari 200320022001

.

28. Diberikan p ≥ 3 bilangan prima dan

{a1, a2, . . . , ap−1} dan {b1, b2, . . . , bp−1}

dua himpunan kelas residu lengkap modulo p. Tunjukkan bahwa

{a1b1, a2b2, . . . , ap−1bp−1}

bukan himpunan kelas residu lengkap modulo p.

29. Tunjukkan bahwa setiap bilanga bulat positif kurang dari n! dapat diny-

atakan sebagai jumlahan tidak lebih dari n bilangan positif yang merupakan

faktor dari n!.

30. Tunjukkan bahwa untuk setiap bilangan ganjil n > 1 berlaku n |2n + 1.

92

31. Diberikan a dan b bilangan bulat positif. Tunjukkan bahwa banyaknya

solusi bulat non-negatif (x, y, z) dari persamaan ax+ by + z = ab adalah

1

2[(a+ 1)(b+ 1) + gcd(a, b) + 1].

32. Diberikan p, q dan r bilangan prima lebih dari 2 dengan sifat qr + 1. Tun-

jukkan bahwa 2r|p− 1 atau p|q2 − 1.

33. Tunjukkan bahwa untuk setiap bilangan bulat positif n berlaku⌊(n− 1)!

n(n+ 1)

⌋.

34. Tentukan semua bilangan bulat positif n sehingga n memiliki kelipatan

dengan digit-digitnya tidak sama dengan 0.

35. Tentukan bilangan bulat terbesar n dengan sifat n habis dibagi oleh setiap

bilangan bulat positif yang kurang dari 3√n.

36. Tentukan semua bilangan real x dengan sifat x⌊x⌊x⌊x⌋⌋⌋ = 88.

37. Tentukan bilangan bulat a, b, c yang sepasang-sepasang relatif prima dengan

a, b, c > 1 dan

b|2a + 1, c|2b + 1, a|2c + 1.

38. Diberikan n bilangan bulat positif dan p1, p2, . . . , pn bilanga prima berbeda

yang lebih dari 3. Tunjukkan bahwa 2p1p2...pn + 1 memiliki setidaknya 4n

faktor positif.

39. Untuk setiap bilangan bulat positif k, didefinisikan p(k) sebagai faktor ganjil

terbesar dari k. Tunjukkan bahwa untuk setiap bilangan bulat positif n

berlaku2n

3<

p(1)

1+

p(2)

2+ . . .+

p(n)

n<

2(n+ 1)

3.

40. Tentukan semua bilangan bulat positif k dengan sifat

τ(n2)

τ(n)= k

untuk suatu bilangan bulat positif n.

93

Materi Pengayaan

1. Dapat di lihat pada website: http://www.imo-official.org

2. Untuk diskusi dengan anak-anak berbakat di bidang matematika silahkan

akses http://www.olimpiade.org

N1. Carilah semua pasangan bilangan bulat (x, y) yang memenuhi persamaan

5x2 + 20x2y2 − 28y2 = 2013.

N2. Apakah terdapat suatu barisan bilangan asli a1, a2, a3, . . . yang memenuhi

dua syarat berikut:

(i) setiap bilangan asli muncul tepat sekali sebagai suku di barisan dan

(ii) untuk setiap bilangan asli n berlaku n habis membagi a1+a2+· · ·+an.

N3. Bilangan asli n dikatakan ”kuat” jika terdapat bilangan asli x sehingga

xnx + 1 habis dibagi 2n.

(a) Tunjukkan bahwa 2013 merupakan bilangan kuat.

(b) Jika n adalah bilangan kuat, tentukan bilangan asli terkecil x sehingga

xnx + 1 habis dibagi 2n.

N4. Bilangan rasional positif x dan y memenuhi persamaan

x+ 2

y + 2+

y + 2

x+ 2= 3− xy.

Buktikan bahwa dalam bentuk pecahan paling sederhana, penyebut dari x

dan y merupakan bilangan kuadrat sempurna.

N5. Cari semua bilangan asli n sehingga terdapat bilangan-bilangan asli x1, x2, . . . , xm

dengan m ≥ 2 yang memenuhi persamaan

x1 + x2 + . . .+ xm = x1x2 . . . xm = n.

94

N6. Suatu himpunan tak kosong A ⊂ Z dikatakan tertutup apabila untuk setiap

a, b ∈ A (tidak harus berbeda) terdapat c ∈ A (boleh sama dengan a atau

b) sedemikian sehingga 2013 | ab + c. Sebagai contoh {0, 1, 2, 3, . . . , 2012}dan {2013} keduanya tertutup, sementara {3, 4, 5, 6, 7} tidak tertutup. No-

tasikan X = {1, 2, 3, . . . , 2013}.

(a) Buktikan bahwa himpunan X dapat dipartisi menjadi tiga himpunan

bagian tak kosong yang masing-masing tertutup.

(b) Buktikan bahwa bagaimanapun cara mempartisi X pada soal (a), se-

lalu terdapat satu partisi dengan banyak unsur paling sedikit 1200.

Catatan. Faktorisasi prima dari 2013 adalah 3 × 11 × 61 dan 1200 =

2× 10× 60 = ϕ(2013) adalah banyak bilangan asli di X yang relatif prima

dengan 2013.

N7. Suatu bilangan asli x dikatakan bersifat ” binom” jika terdapat bilangan-

bilangan asli m dan n dengan 1 < m ≤ n2sehingga

(nm

)= x.

(a) Buktikan bahwa 2013 tidak bersifat binom.

(b) Apakah ada bilangan asli kelipatan 2013 yang bersifat binom?

(Keterangan:(nm

)= n!

(n−m)!m!)

N8. Suatu fungsi f : N → N dikatakan komutatif dengan FPB apabila berlaku

f(gcd(m,n)) = gcd(f(m), f(n))

untuk setiap bilangan asli m,n dan dikatakan multiplikatif apabila berlaku

f(mn) = f(m)f(n)

untuk setiap bilangan asli m,n.

(a) Buktikan bahwa terdapat fungsi multiplikatif yang tidak komutatif

dengan FPB.

(b) Buktikan bahwa fungsi multiplikatif yang juga memenuhi f(f(n)) = n

untuk setiap bilangan asli n selalu komutatif dengan FPB.

95

N9. Pasangan bilangan bulat positif (p, q) dikatakan serasi jika p dan q kedu-

anya prima atau relatif prima. Tentukan semua pasangan serasi (p, q) yang

memenuhi sifat terdapat bilangan bulat non negatif n sehingga

p+ q

pq=

n− 2

n2 + 4

Solusi

Hubungi Pembina Tim IMO Indonesia

96